exam 3

Ace your homework & exams now with Quizwiz!

A client complains of pain. Which question asked by the nurse are most appropriate to assess the nature of the pain?

"Can you describe your pain to me?" R: The nurse may ask the client to describe the pain or to point the area that hurts. It may help to assess the nature of the pain. Asking about effect on pain with movement may help to assess precipitating factors. The severity of a pain could be identified by asking the client to rate it on a scale from 0 to 10. The precipitating factors can be identified by asking the client about worsening of the pain with a particular activity.

A client arrives at the clinic and tells the nurse about various aches and pains since her spouse's death 3 months ago. The client appears depressed and tense. What is the initial nursing intervention? Correct1 Facilitating a discussion of the spouse's death 2 Focusing on teaching the client relaxation exercises 3 Asking the practitioner for a psychiatric consultation 4 Helping the client recognize ambivalence toward the spouse

1

A client is diagnosed with parathyroid dysfunction. Which serum calcium concentration in the client would support the diagnosis? Correct1 7.8 mg/dL 2 8.9 mg/dL 3 9.7 mg/dL 4 10.2 mg/dL

1

What response from the nurse demonstrates an understanding of hallucinating behavior by a client? Correct1 Asking, "What are the voices telling you to do?" 2 Calmly noting that the "rat on the floor" is really a stuffed toy 3 Allowing the family to bring prepackaged foods from the store 4 Explaining to the family that the behavior will worsen during the night

1

Which condition is characterized by hemorrhage after a pregnant female delivers? Correct1 Sheehan's syndrome 2 Cushing's syndrome 3 Addison's syndrome 4 Schwartz-Bartter syndrome

1

A client is found to have a mood disorder, hypomanic episode. To support the diagnosis, the nurse should identify the signs and symptoms associated with this disorder. Select all that apply. Correct1 Distractibility Correct2 Flight of ideas 3 Low self-esteem 4 Increased need for sleep 5 Psychomotor retardation

1,2

A client is admitted to the hospital with a diagnosis of restrictive airway disease. The nurse expects the client to exhibit which early signs of respiratory acidosis? Select all that apply. Correct1 Headache Correct2 Irritability Correct3 Restlessness 4 Hypertension 5 Lightheadedness

1,2,3

Which drug would be effective for the treatment of pituitary Cushing's syndrome? Incorrect1 Mitotane 2 Cabergoline Correct3 Cyproheptadine 4 Bromocriptine mesylate

3

A Nitrazine test strip that turns deep blue indicates that the fluid being tested has a pH of what? 1 4.5 2 5.5 3 6.5 Correct4 7.5

4

The nurse was assessing an elderly client and recorded the pulse rate as 85. After assessment the nurse determined the cardiac output as 5950. What could be the approximate stroke volume?

70 mL.

The nurse caring for a client with a systemic infection is aware that the assessment finding that is most indicative of a systemic infection is what? Select all that apply.

An elevated temperature of 101.3° F (38.5° C) is most indicative of a systemic infection. A WBC count of 8200/mm3 (8.2 X 109/L) is within the normal range of 5000 to 10,000/mm3 (5 to 10 X 109/L). Pedal edema is generally not related to an infectious process. Pale skin and nail beds may be related to an infectious process, but not necessarily.

What important step should the nurse take to evaluate lifestyle change in the client?

Encourage the client to maintain an exercise and eating calendar to track adherence and provide positive reinforcement

Cheyne-Stokes respiration

In Cheyne-Stokes respiration, a client's breathing pattern is characterized by *progressively deeper and faster breathing*, that is, *hyperventilation* followed by *apnea*.

A nurse applies an ice pack to a client's leg for 20 minutes. The cold application will cause what physiologic effect?

Local anesthesia R: Cold reduces the sensitivity of pain receptors in the skin. In addition, local blood vessels constrict, limiting the amount of edema and its related pressure and discomfort. Local blood vessels constrict. Local cold applications do not depress vital signs. Local cold applications do not affect blood viscosity directly.

A nursing student is recalling the definition of Nurse Practice Acts. What do the Nurse Practice Acts do?

Nurse Practice Acts describe and define the legal boundaries of nursing practice within each state

The nurse is caring for a client who had a hip replacement 2 days prior. After removing a bedpan from under the client, what is a priority nursing intervention?

Provide perineal care

Test-Taking Tip: Make certain that the answer you select is reasonable and obtainable under ordinary circumstances and that the action can be carried out in the given situation.

Test-Taking Tip: Make certain that the answer you select is reasonable and obtainable under ordinary circumstances and that the action can be carried out in the given situation.

Which act protects a person who is HIV positive?

The The Americans with Disabilities Act (ADA)

A nurse and a nutritionist are discussing the needs of a client who practices the Russian Orthodox faith. What should the nurse and the nutritionist consider when planning meals for this client?

The client does not eat meat on Wednesdays and Fridays. A client who is Russian Orthodox may refrain from eating meat on Wednesdays and Fridays, and the nurse and the nutritionist should consider this when developing a nutrition plan for the client. A client who is Jewish might avoid pork and shellfish as part of a kosher diet. A client who practices the Jehovah's Witness faith might avoid blood-containing food. A client who is Hindu might follow a strict vegetarian diet.

Which belief of a nurse may have a negative effect on health services for minority clients?

The mind, body, and spirit are distinct entities. Eastern tradition considers body-mind-spirit as a single entity; if the nurse refuses to accept and respect that a client of a different culture may believe this, then the quality of care can be affected. Children of multiracial, multicultural, and multiheritage marriages fall into more than one category, so the nurse should focus on culture rather than race. For the Chinese, disease is caused by fluctuations in opposing forces—the yin-yang energies. The nurse should not make assumptions about a client's race based on his or her appearance; the nurse should record the client's race as he or she self-identifies.

A nurse is preparing to administer an oil-retention enema and understands that it works primarily by doing what?

The primary purpose of an oil-retention enema is to *lubricate the sigmoid colon and rectum*. Secondary benefits of an oil-retention enema include stimulating the urge to defecate and softening feces. An oil-retention enema does not dissolve feces.

What is an example of the critical thinking attitude of independent thinking in nursing practice?

To talk with other nurses to share ideas about nursing intervention R: Nurses talking to each other and sharing ideas about nursing interventions reflects independent thinking. Responsibility and authority require referring to a policy and procedure manual for reviewing steps of a skill. Humility involves recognizing the need for more information for making a decision. Curiosity is exploring and learning about clients to help make appropriate clinical judgment.

Which definition is involved in the caring process called knowing according to Swanson's theory of caring?

Trying to understand an event that has meaning in the life of another.

A client has a stage III pressure ulcer. Which nursing intervention can prevent further injury by eliminating shearing force?

Use draw sheets to pull up, transfer and position the client

Which are examples of high-reliability organizations? Select all that apply.

Which are examples of high-reliability organizations? R: Aviation, air traffic control, and nuclear power plants are examples of high-reliability organizations because they have to operate in hazardous conditions yet have very few adverse events. Cancer hospitals and state transportation agencies are not high-reliability organizations because they do not operate under hazardous conditions.

Which statement regarding breast cancer stage and mortality is correct?

Which statement regarding breast cancer stage and mortality is correct? African American and Puerto Rican women have the highest risk for triple negative breast cancer. Caucasian women older, not younger than 40 years old are at a greater risk than other racial/ethnic groups. Breast cancer death rates are highest in African American, Hawaiian, Puerto Rican, and Samoan women. Non-Hispanic white women are more likely to present with an earlier-stage breast cancer than are American Indians/Alaskan Native, Asian Indian/Pakistani, black, Filipino, Hawaiian, Mexican,Puerto Rican, and Samoan women.

The nurse finds that a client with bilateral oral swelling, pain, and trismus had undergone a surgical extraction of an impacted tooth five days ago. What type of nursing diagnosis does the documentation of acute pain refer to?

actual

A client complains to the nurse that a staff member did not respond to the client's call. The nurse politely reassures the client, and makes the client comfortable. The nurse speaks to the staff member about the incident and solves the problem. Which critical thinking attitude has the nurse demonstrated in this situation?

fairness - listening to both sides

Cardiac output is obtained by

multiplying the heart rate and the stroke volume.

A client sustains a crushing injury to the lower left leg, and a below-the-knee amputation is performed. For which common clinical manifestations of a pulmonary embolus should the nurse assess this client? Select all that apply. Correct1 Sharp chest pain Correct2 Acute onset of dyspnea 3 Pain in the residual limb 4 Absence of the popliteal pulse 5 Blanching of the affected extremity

1,2

What are the functions of a client's subcutaneous layer of skin? Select all that apply. Correct1 It provides insulation. Correct2 It acts as an energy reservoir. 3 It prevents systemic dehydration. Incorrect4 It provides cells for wound healing. Correct5 It acts as a mechanical shock absorber.

1,2,5

Which does the nurse understand related to negative pressure wound therapy? Select all that apply. Correct1 Using a suction pump 2 Treating necrotizing infections 3 Administering oxygen under high pressure 4 Application of a low-voltage current to a wound area Correct5 Reducing chronic ulcers by removing fluids from the wound

1,5

What is the correct order when solving an ethical dilemma?

1. Collect relevant case-related information. 2. Clarify values. 3. Verbalize the problem 4. Determine possible courses of action 5. Negotiate a plan 6. Evaluate the plan over time

What factors put a client at risk for bacterial infections? Select all that apply. 1 Dry skin 2 Underweight Correct3 Atopic dermatitis Correct4 Diabetes mellitus Correct5 Systemic antibiotics

3,4,5

1 Freedom from injury 2 Engaging independently in solitary craft activities 3 Identifying the consequences of acting-out behavior Correct4 Interacting appropriately with others in the therapeutic milieu

4

A 3-year-old child is found to have a pervasive developmental disorder not otherwise specified. What should the nurse consider most unusual for the child to demonstrate? 1 Interest in music 2 Ritualistic behavior 3 Attachment to odd objects Correct4 Responsiveness to the parents

4

A client who has been admitted with a diagnosis of schizophrenia says to the nurse, "Yes, it's March. March is Little Women. That's literal, you know." What do these statements illustrate? 1 Echolalia 2 Neologisms 3 Flight of ideas Correct4 Loosening of associations

4

A 5-year-old with attention deficit-hyperactivity disorder (ADHD) exhibits a short attention span and demonstrates intermittent head-banging and hair-pulling, as well as excessive motor activity. What is the priority nursing objective for this child? 1 Facilitating sleep Correct2 Maintaining safety 3 Promoting body image 4 Increasing nutritional intake

2

A client with the diagnosis of an antisocial personality disorder responds to limit-setting by a nurse by saying, "You sure do look messy today." What is the most appropriate response by the nurse? 1 "Don't you feel well today?" Correct2 "I get the feeling you're angry with me." 3 "I really didn't think anyone would notice." Incorrect4 "Do you think that was a nice thing to say to me?"

2

How should nurses provide effective nursing care to clients from different cultural backgrounds? 1 By advising clients that some cultural practices may be harmful to health Correct2 By providing care that fits the clients' cultural beliefs 3 By strictly adhering to organization policies regarding nursing care 4 By ignoring the cultural aspect and focusing on the medical aspect of care

2

Which change in the epidermis causes increased risk of sunburn? 1 Decreased cell division Correct2 Decreased melanocyte activity 3 Decreased vitamin D production 4 Decreased immune system cells

2

A nurse at the mental health center has been counseling the family of an adolescent client with anorexia about nutrition. Which statement made by a family member demonstrates an adequate understanding of the needs of the client? 1 "We won't have to worry about this passing fad for long." 2 "We'll monitor both of our teenagers' exercise habits closely." 3 "We need to watch more closely when we're all eating together." Correct4 "We should give our child more input into our meal planning."

4

A client is scheduled for an adrenalectomy. What does the nurse expect that the plan of care will include? 1 Low-protein diet Correct2 Parenteral corticosteroids 3 Preoperative 24-hour urine specimen 4 Withholding all medications 48 hours before surgery

2

Which surgery is used to treat excessive wrinkling or sagging of facial skin? 1 Rhinoplasty Correct2 Rhytidectomy 3 Dermabrasion 4 Blepharoplasty

2

Which sexually transmitted disease is treated with antiviral drugs? 1 Syphilis 2 Gonorrhea Correct3 Genital herpes 4 Chlamydial infection

3

A client experiencing a tremendously stressful situation says, "My baby was diagnosed with terminal cancer 2 months ago. I'm either crying or walking around like I'm in a dream. I can't believe this is happening. What did we do to deserve something so horrible? The doctors can transplant almost every human organ, but they can't stop my baby from dying. I'm so angry. Most days I just want to take my child and run away." The nurse determines that the client is mainly expressing what? 1 Anger 2 Denial 3 Avoidance Correct4 Anticipatory grief

4

A nurse manager works on a unit where the nursing staff members are uncomfortable taking care of clients from cultures that are different from their own. How should the nurse manager address this situation? 1 Assign articles about various cultures so that they can become more knowledgeable. 2 Relocate the nurses to units where they will not have to care for clients from a variety of cultures. 3 Rotate the nurses' assignments so they have an equal opportunity to care for clients from other cultures. Correct4 Plan a workshop that offers opportunities to learn about the cultures they might encounter while at work.

4

What concept of death should a nurse expect a 4-year-old child to have? 1Cessation of life 2Reversible separation 3Only affects old people 4Force takes one away from family

2Reversible separation

An older client whose family has been visiting the client in the psychiatric unit is visibly angry and says to the nurse, "My daughter-in-law says they can't take me home until the doctor lets me go. She doesn't understand how important this is to me; she's not from our culture." What should the nurse do? 1Ignore the statement for the present. 2Say, "You feel she doesn't want you at home." 3Reflect on the client's feelings about the cultural differences. 4Respond, "The doctor is the one who makes decisions about discharge."

2Say, "You feel she doesn't want you at home."

A 15-year-old client is being assessed in the adolescent clinic. He has a history of drug abuse, stealing, refusing to comply with rules, and demonstrating an inability to get along with others in any setting. When obtaining the health history, the nurse may be prevented from accurately listening to what the client is saying because of what? 1The client's disease process 2The nurse's personal cultural beliefs 3The pressure of time to complete care 4The personal need to secure information

2The nurse's personal cultural beliefs

A client is receiving whole-body radiation for Hodgkin disease. Which side effect should the nurse expect as a result of this therapy? 1 Increased tendency to bleed 2 Increased tendency for fractures Correct3 Decreased number of erythrocytes 4 Decreased susceptibility to infection

3

A client who has a hemoglobin of 6 gm/dL (60 mmol/L) is refusing blood because of religious reasons. What is the most appropriate action by the nurse? 1 Call the chaplain to convince the client to receive the blood transfusion. 2 Discuss the case with coworkers. Correct3 Notify the primary healthcare provider of the client's refusal of blood products. 4 Explain to the client that they will die without the blood transfusion.

3

A client who has had a subtotal thyroidectomy does not understand how hypothyroidism can develop when the problem was initially hyperthyroidism. On what fact should the nurse base her response? 1 Hypothyroidism is a gradual slowing of the body's function. 2 There will be a decrease in pituitary thyroid-stimulating hormone (TSH). Correct3 There may not be enough thyroid tissue to supply adequate thyroid hormone. 4 Atrophy of tissue remaining after surgery reduces secretion of thyroid hormones.

3

the pressure exerted on the skin when a debilitated client is pulled up in bed without a draw sheet or when the client slides down in bed.

Shearing Force With shearing, the skin adheres to the bed linens while the layers of subcutaneous tissue and bone slide in the direction of the body movements, causing tearing of the skin. *Using a draw sheet can reduce and minimize friction and shearing force*.

A healthcare provider prescribes a diuretic for a client with hypertension. What should the nurse include in the teaching when explaining how diuretics reduce blood pressure? 1 Facilitates vasodilation 2 Promotes smooth muscle relaxation Correct3 Reduces the circulating blood volume 4 Blocks the sympathetic nervous system

3

A married woman is brought to the emergency department of a local hospital. Her eyes are swollen shut, and she has a bruise on her neck. She reports that she is being beaten by her husband. How does the nurse expect the husband to behave when he arrives at the emergency department? 1 Fearful 2 Confused Correct3 Charming 4 Indifferent

3

The nurse is caring for an African American client with renal failure. The client states that the illness is a punishment for sins. Which cultural health belief does the client communicate? 1Yin/Yang balance 2Biomedical belief 3Determinism belief 4Magicoreligious belief

4 Magicoreligious belief

Limiting head of bed elevation, repositioning, and range of motion are interventions that may prevent

pressure related injury verses shear injury.

A client who is in a late stage of pancreatic cancer intellectually understands the terminal nature of the illness. What are behaviors that indicate the client is emotionally accepting the impending death?

*Revising the will and planning a visit to a friend are realistic, productive, and constructive ways of using this time* Crying and talking openly about death are signs of depression. Going from healthcare provider to healthcare provider demonstrates disbelief, denial, or desperation. Refusing to follow treatments and stating that the client is going to die anyway indicates anger and hopelessness, not acceptance.

What are common negligent acts of nurses found in the hospital setting? Select all that apply.

- Failure to notify the healthcare provider of problems - Failure to follow the six rights of medication administration - Failure to ensure the safety of a client with disequilibrium problems

A client is admitted to the mental health clinic for treatment of an obsessive-compulsive disorder that impairs ability to work outside the home. What should the nurse consider about the client's behavior when developing a plan of care? Correct1 The client knows that the behavior is illogical but is unable to stop it. 2 The client is probably unaware that the behavior is inappropriately repetitious. 3 This behavior is partially determined by the culture to which the client belongs. 4 This behavior can be stopped by having the nursing staff gently encourage the client.

1

A nurse, understanding the possible cause of alcohol-induced amnestic disorder, should take into consideration that the client is probably experiencing which imbalance? Correct1 Thiamine deficiency 2 A reduced iron intake 3 An increase in serotonin 4 Riboflavin malabsorption

1

In which situation does the nurse consider the family as context? Correct1 The nurse is caring for an individual with tonsillitis. 2 The nurse is caring for a dying client and all the family members. 3 The nurse is teaching young parents about caring for their toddler. Incorrect4 The nurse is assessing the needs of the family caregivers of a client.

1

Which drug is a newer treatment option for treating metastatic melanoma? 1 Lomustin Correct2 Ipilimumab 3 Carmustine 4 Temozolomide

2

Which of these cultural groups adopts a combination of dietary, herbal, and other naturalistic therapies to prevent and treat illness? 1 East Asian 2 Hispanic Correct3 Asian Indian 4 Native American

3

Why is it important for a nurse in the prenatal clinic to provide nutritional counseling to all newly pregnant women?

Different cultural groups favor different essential nutrients. The nurse should become informed regarding the cultural eating patterns of clients so that foods containing the essential nutrients that are part of these dietary patterns may be included in the diet. Fluid retention is only one component of weight gain; growth of the fetus, placenta, breasts, and uterus also contributes to weight gain. The need for calories and nutrients increases during pregnancy. Pregnancy diets are not specific; they are composed of the essential nutrients.

A client with dementia who feels highly anxious and confused believes that the current day is actually different than what it is. Which statement made by the nurse is an example of validation therapy?

Yes, today is the day that you just mentioned

Which statement should the nurse include when providing anticipatory guidance to the parents of a 3-year-old client? Correct1 "It is important to set limits with your child." 2 "Your child may begin to have more nightmares." 3 "It is important to enroll your child in swimming lessons." 4 "Your child may begin to exhibit more aggressive behavior."

1

Which type of hormonal imbalance does the nurse infer through this image? old woman pic from 50's and current Correct1 Acromegaly 2 Exophthalmos 3 Addison's disease Incorrect4 Cushing's syndrome

1

Which physical changes may cause longitudinal nail ridges? Incorrect1 Decreased rate of growth 2 Decreased cell division Correct3 Decreased blood flow 4 Decreased vitamin D production

3

The nurse is explaining behavior to a nursing assistant. Which characteristic of a behavior usually results in that behavior being viewed and accepted as normal? 1Fitting within standards accepted by one's society 2Helping the person reduce the need for coping skills 3Expressing the individual's feelings and thoughts accurately 4Allowing achievement of short-term and long-term goals by the individual

1Fitting within standards accepted by one's society

A newly immigrated older Chinese adult is brought to a mental health clinic when family members become concerned that their parent is depressed. In an attempt to conduct a culturally competent assessment interview, the nurse asks certain questions. Which questions does the nurse ask? Select all that apply. 1 "What brought you here for treatment today?" 2 "What do you believe is the cause of your depression?" 3 "Does religion have a role in your perception of health and wellness?" 4 "Do you have insurance that includes coverage of mental health issues?" 5 "Have you ever sought treatment for a mental health problem previously?"

1 "What brought you here for treatment today?" 2 "What do you believe is the cause of your depression?" 3 "Does religion have a role in your perception of health and wellness?" 5 "Have you ever sought treatment for a mental health problem previously?"

Since giving birth 6 months ago, a new mother has breast-fed her infant. The woman becomes hysterical after learning that her husband has been seriously injured in an automobile accident. Culturally this woman believes that emotional stress while breast-feeding can "sour the milk," and she indicates that she must wean her infant immediately. What should the nurse do? 1 Instruct the mother about formula feeding. 2 Explain to the mother that these beliefs are wrong. 3 Provide the mother with books indicating that the milk does not sour. 4 Encourage the mother to take an anti-anxiety drug while continuing breast-feeding

1 Instruct the mother about formula feeding

Alternative therapy measures have become increasingly accepted within the past decade, especially in the relief of pain. Which methods qualify as alternative therapies for pain? Select all that apply. 1 Prayer 2 Hypnosis 3 Medication 4 Aromatherapy 5 Guided imagery

1 Prayer 2 Hypnosis 4 Aromatherapy 5 Guided imagery

Which internal variable influences health beliefs and practices? 1 Family practices 2 Cultural background 3 Socioeconomic factors 4 Intellectual background

4 Intellectual background Rationale: Intellectual background is an internal factor that affects the client's health beliefs and practices. A client's knowledge, educational background, and past experiences influence how a client thinks about health. Family practices, cultural background, and socioeconomic factors are among the external factors that influence health beliefs and practices.

What does a public health nurse expect to encounter when working with families raised in a culture of poverty? 1 Willingness to delay gratification 2 Optimism about improving their lifestyle 3 Shame because of their inadequacy as parents 4 Powerlessness relative to changing their situation

4 Powerlessness relative to changing their situation

Which carative factor is involved in creating a healing environment at all levels, physical and non-physical, according to Watson's Transpersonal Caring? 1Promoting transpersonal teaching-learning 2Promoting and expressing positive and negative feelings 3Developing a helping, trusting, human caring relationship 4Providing for a supportive, protective, and/or spiritual environment

4 Providing for a supportive, protective, and/or spiritual environment

The parents of an autistic child begin family therapy with a nurse therapist. The father states that the family members wish to share their religious beliefs with the therapist. What should the nurse do? 1Limit the father's discussion of religion. 2Include the mutual discussion of religious beliefs. 3Invite the family's religious leader to a therapy session. 4Encourage family discussion of their religion in the sessions

4Encourage family discussion of their religion in the sessions

A 15-year-old emancipated minor gave birth to a boy 36 hours ago and has requested a circumcision. What is the medical priority?

Obtaining an informed consent signed by the mother of the baby As an emancipated minor, the mother of the baby has the right to make the decision regarding the circumcision and is responsible for signing the informed consent. A nurse does not administer a lidocaine injection into the newborn penis; the physician does. Circumcision is a decision made by the parent(s), often for cultural or religious reasons. Education can be provided as needed, as the provider obtains a signed consent. Because the mother of the baby is an emancipated minor, the grandmother does not have the authority to sign the consent in this situation.

The nurse is caring for an Asian-American client with a diagnosis of depression. While interviewing this client the nurse notes that the client maintains traditional cultural beliefs and values. What is the most important information for the nurse to obtain about the client? Dietary practices Concept of space Immigration status Role within the family

Role within the family Rationale If an Asian-American client adheres to traditional Asian practices, the nurse must recognize that the family is the central and most important social force acting on the individual. Dietary practices, concept of space, and immigration status are not as significant as family dynamics.

A nurse needs to educate a parent about the playing behavior of preschoolers. What information should the nurse provide to the parent? 1 "Your child will avoid playing different roles." 2 "Your child will dislike having a leader for any activity." Correct3 "Your child will cooperate while playing with another child." 4 "Your child will often like to play with a group of 7-9 children."

3

A nurse on a psychiatric unit has been working with a suicidal college student for 2 days. What comment by the student indicates relief from suicidal thinking? 1 "I can be a burden to others." 2 "I feel very alone sometimes." Correct3 "I plan to go to school next semester." 4 "I don't know whether I can talk about my feelings."

3

A client says, "None of the medications will work on me because I am away from my holy land." What course of action should the nurse take to comply with teamwork and collaboration competency according to the Quality and Safety Education for Nurses (QSEN)? 1 Provide care to the client with respect to his or her diversity, values and beliefs 2 Approach the agency chaplain to discuss the spiritual needs of the client 3 Conduct thorough research on the effect of emotional distress on the client's health 4 Use the flow chart data to provide the best care and monitor the outcome of care processes

-2 Approach the agency chaplain to discuss the spiritual needs of the client

A recent immigrant from mainland China is critically ill and dying. What question should the nurse ask when collecting information to meet the emotional needs of this client? 1"Do you like living in this country?" 2"When did you come to this country?" 3"Is there a family member who can translate for you?" 4"Which family member do you prefer to receive information?"

-4"Which family member do you prefer to receive information?"

A client who uses a ritual of counting paper in the printer tells the nurse, "I'm spending 30 minutes counting each time I make copies, and my boss is getting very upset. What should I do?" What is the best response by the nurse? Correct1 "Limit photocopying by clustering it to two or three times a day." 2 "Arrive at work 30 minutes early to count the paper in the printer." 3 "Substitute another activity at home, such as counting shoes or other objects." 4 "Talk with the boss to ask for tolerance until the mental health treatments help."

1

A client with a prolonged history of chronic schizophrenia, paranoid type, shows the nurse a small plastic keychain and says that it provides protection from evil forces. The client then quickly hides the keychain, yelling, "Don't take it away from me; it's the only thing that protects me." How should the nurse respond? Correct1 "You may keep it, because I know it's important to you." 2 "You need to give it to me, because you may hurt yourself." 3 "You're safe without the keychain, because there are no evil forces here." 4 "You'd better put it away, because someone might take it away from you."

1

A 65-inch (165 cm) tall 15-year-old girl weighing 80 lb (36.3 kg) is admitted to a mental health facility with a diagnosis of anorexia nervosa. The nurse recognizes that her problem most likely is caused by what? Correct1 A desire to control her life Incorrect2 The wish to be accepted by her peers 3 The media's emphasis on the beauty of thinness 4 A delusion in which she believes that she must be thin

1

A client reports drinking two drinks per day every day with no negative consequences. How should this person be classified? Correct1 Daily drinker 2 Substance abuser 3 Functional alcoholic 4 Substance dependent

1

A client who sustained a burn injury involving 36% of the body surface area is receiving hydrotherapy. Which is the best nursing intervention when providing wound care? Correct1 Use a consistent approach to care and encourage participation. 2 Prepare equipment while doing the procedure and explain the treatment to the client. 3 Rinse the burn area with 105° F (40.6° C) water to prevent loss of body temperature. 4 Arrange for a change of staff every 4 to 5 days and have the client select the time for the procedure to be done.

1

As the nurse is discussing psychiatric care with an older adult client, the client says, "When I was growing up I was taught to accept my lot in life and not complain. I'm proud of the fact that despite my issues I can still function independently. I don't want to be just put away." The nurse understands that the factors that influence the client's mental health are examples of what? 1 Setting of care 2 Anxiety disorder Correct3 Attitudes and beliefs 4 Cultural and ethnic disparities

3

At times a client's anxiety level is so high that it blocks attempts at communication and the nurse is unsure of what is being said. To clarify understanding, the nurse says, "Let's see whether we mean the same thing." What communication technique is being used by the nurse? 1 Reflecting feelings 2 Making observations Correct3 Seeking consensual validation 4 Attempting to place events in sequence

3

How can a nurse minimize agitation in a disturbed client? 1 By ensuring constant staff contact 2 By increasing environmental sensory stimulation Correct3 By limiting unnecessary interactions with the client 4 By discussing the reasons for the client's suspicions

3

One afternoon the nurse on the unit overhears a young female client having an argument with her boyfriend. A while later the client complains to the nurse that dinner is always late and the meals are terrible. The nurse identifies the defense mechanism that the client is using as what? 1 Projection 2 Dissociation Correct3 Displacement 4 Intellectualization

3

A male client with the diagnosis of schizophrenia, paranoid type, often displays overt sexual behavior toward female clients and nurses. What is the nurse's best response when the client engages in sexually explicit behavior? 1 Refusing to speak with the client until he stops the behavior 2 Sending the client to his room when the behavior is observed 3 Ignoring this behavior until the client is more in control of his responses Correct4 Telling the client in a matter-of-fact manner that his behavior is unacceptable

4

A nurse in a hospice program cares for clients and family members who are coping with imminent loss. What is the most important factor in predicting a person's potential reaction to grief? 1 Family interactions 2 Social support system 3 Emotional relationships Correct4 Earlier experiences with grief

4

A nurse is assessing the grief response of a family member whose relative has died. What must the nurse consider first about the family to conduct an effective assessment?

Cultural background The degree of anguish experienced or expressed is most often set or imposed by the cultural background of the individual, so cultural background must be assessed before care is planned. Although personality traits do enter into the grief process, they are not as important in the developing awareness stage as is cultural background. Educational level has no relationship to the grieving process; nor does socioeconomic class.

The nurse is providing postoperative care to a client who had a submucosal resection (SMR) for a deviated septum. The nurse should monitor for what complication associated with this type of surgery?

Expectoration of blood After an SMR, hemorrhage from the area should be suspected if the client is swallowing frequently or expelling blood with saliva. A headache in the back of the head is not a complication of a submucosal resection. Crepitus is caused by leakage of air into tissue spaces; it is not an expected complication of SMR. The nerves and structures involved with speech are not within the operative area. However, the sound of the voice is altered temporarily by the presence of nasal packing and edema.

A client who has a hemoglobin of 6 gm/dL (60 mmol/L) is refusing blood because of religious reasons. What is the most appropriate action by the nurse? Call the chaplain to convince the client to receive the blood transfusion. Discuss the case with coworkers. Notify the primary healthcare provider of the client's refusal of blood products. Explain to the client that they will die without the blood transfusion.

Notify the primary healthcare provider of the client's refusal of blood products. Rationale The nurse serves as an advocate for the client to uphold their wishes. Synthetic blood products are available but must be prescribed by the primary healthcare provider. Therefore the primary healthcare provider needs to be notified of the client's refusal for blood so alternatives can be considered. The chaplain's role is to offer support, not to convince the client to go against beliefs. It is a Health Insurance Portability and Accountability Act (HIPAA) (Canada: Personal Health Information Protection Act [PHIPA]) violation to discuss the case with coworkers unless they are involved in the care of the client. The nurse should not use threats or fear to coerce the client.

used as an objective measurement of consciousness on a numerical scale. This scale assigns a numeric score for each area of neurologic status of the client. A higher score equates to a higher neurologic function.

The Glasgow Coma Scale (GCS) is

A client says "Do not cut the thread on my wrist before sending me for surgery because the thread is a blessing from God." Which internal variable influences the client's health belief in this scenario? 1Spiritual factors 2Emotional factors 3Developmental stage factors 4Intellectual background factor

1Spiritual factors

A client with schizophrenia is started on a regimen of chlorpromazine. After 10 days a shuffling gait, tremors, and some rigidity are apparent. Benztropine mesylate 2 mg by mouth daily is prescribed. What does the nurse remember when administering these medications together? 1 Both medications are cholinesterase inhibitors. Correct2 Both medications have a cholinergic-blocking action. 3 The antipsychotic effects of chlorpromazine will be decreased. 4 The synergistic effect of these medications will cause drooling.

2

A client with a history of gambling is experiencing legal difficulties for embezzling money and has been required to obtain counseling. During an intake interview the client says, "I never would have done this if I'd been paid what I am worth." What factor will create the greatest difficulty in helping this client develop insight? 1 Feelings of boredom and emptiness 2 Grandiosity related to personal abilities Correct3 Projection of reasons for difficulties onto others 4 Anger toward those who are in authority positions

3

A client with a long history of alcohol abuse who has been hospitalized for 1 week tells the nurse, "I feel much better and probably won't need any more treatment." What does the nurse conclude when evaluating the client's progress? 1 The client has accepted the illness and now must use willpower to resist alcohol. 2 The client will probably not use alcohol again as long as the client's family remains supportive. Correct3 The client's lack of insight into the emotional aspects of the illness indicates the need for continued supervision. 4 The client's statement should be communicated to the practitioner so aversion therapy can be started before the client's discharge.

3

A primigravida has just given birth. The nurse is aware that the client has type AB Rh-negative blood. Her newborn's blood type is B positive. What should the plan of care include? 1 Determining the father's blood type 2 Preparing for a maternal blood transfusion 3 Observing the newborn for signs of ABO incompatibility Correct4 Obtaining a prescription to administer Rho(D) immune globulin to the mother

4

Which internal variable influences health beliefs and practices? 1 Family practices 2 Cultural background 3 Socioeconomic factors 4 Intellectual background

4

The nurse is providing care to a bilingual preschool-age child. Which should the nurse take into consideration regarding the child's language development? Select all that apply.

Disabilities will manifest in both languages. Milestones are reached at the same stage for both languages. When planning care for a bilingual preschool-age child, the nurse should consider that language disabilities will manifest in both languages and that milestones for both languages will be reached during the same time frame. There is no evidence that the language spoken in the home will be less developed. A bilingual child will be able to read in both languages. Bilingual children should not act as medical interpreters for their family members.

Which theory proposes that older adults experience a shift from a materialistic to cosmic view of the world?

Gerotranscendence theory The gerotranscendence theory is a recent theory that proposes that the older adult experiences a shift in perspective with age. The person moves from a materialistic and national view of the world to a more cosmic and transcendent one. The activity theory considers the continuation of activities performed during middle age as necessary for successful aging. The continuity theory suggests that a person's personality remains stable and behavior becomes more predictable as people age. The disengagement theory states that aging individuals withdraw from customary roles and engage in more introspective, self-focused activities.

The nurse expects a client with an elevated temperature to exhibit what indicators of pyrexia? Select all that apply.

Increased body heat dilates blood vessels, causing a flushed face. The pulse rate increases to meet increased tissue demands for oxygen in the febrile state. Fever may not cause difficult breathing. Pain is not related to fever. Blood pressure is not expected to increase with fever.

The nurse is advised to join a community health center that mainly caters to Latino clients. Which skills should the nurse develop to help reduce health disparities? Select all that apply.

Learning to speak basic medical Spanish Learning about the health literacy rate of the community Incorporating the health beliefs of the community in any nursing care plans Learning about and respecting unique beliefs and values prevalent among the group In order to provide effective health care service to the ethnic group, the nurse should learn to speak basic medical Spanish. This promotes communication and develops trust between the nurse and the clients. Learning about the clients' health literacy can help the nurse identify areas of opportunity for client education and health promotion. Incorporation of beliefs and values in plans of care can make the care more effective. The nurse should learn about the unique values and beliefs of the ethnic group and respect them to deliver equitable health care. Updating the clinical supplies at the health care facility is a basic responsibility of the nurse, but it will not help reduce health disparity.

A nurse understands that when a client is a member of a different ethnic community it is important to do what? Ensure that the nurse's biases are understood by the family. Make plans to counteract the client's misconceptions about therapies. Offer a therapeutic regimen compatible with the lifestyle of the family. Recognize that the client's responses will be similar to other clients' responses.

Offer a therapeutic regimen compatible with the lifestyle of the family. Rationale The client cannot be expected to accept or even respond to a plan that is incompatible with the family's lifestyle. The family should not have to adjust to the nurse's biases; the nurse must self-identify biases and ensure that they do not interfere with nursing care. There is no evidence that misconceptions will occur. All individuals respond differently to situations.

When caring for a client who adheres to a kosher diet, which important thing should the nurse make sure to exclude from the client's meals?

Pork and shellfish A client who adheres to a kosher diet will not eat pork and shellfish. Jehovah's Witnesses will not eat blood-containing food products. Some Buddhist and Hindu clients may be vegetarians and avoid all meat, fish, and poultry. Russian Orthodox clients will avoid animal and dairy products during Lent.

A nurse is performing health screenings of toddlers in a culturally diverse neighborhood. Which child should the nurse consider at risk for beta-thalassemia (Cooley anemia)?

Two-year-old child of Greek descent with a large abdomen Beta-Thalassemia is common in children who are black or of Mediterranean descent (Italian, Greek, Syrian); an enlarged abdomen may be the result of hepatomegaly or splenomegaly. Pale skin is expected in children of Irish descent; children with β-thalassemia may have bronze skin as a result of hemosiderosis if the excess iron is not chelated. Defective hemoglobin leads to damaged red blood cells and a decreased hematocrit. Asian descent is not a risk factor for β-thalassemia.

A 30-year-old woman is scheduled for a total abdominal hysterectomy because of noninvasive endometrial cancer. The nurse anticipates the client may have difficulty adjusting emotionally to this type of surgery. What is the most common reason for this difficulty? Correct1 Loss of femininity 2 Body image changes 3 Diminished sexual desire 4 Slow postmenopausal recovery

1

An African man presents to the emergency department to obtain pain medication. The nurse behaves judgmentally and labels the client a drug abuser. What is the nurse demonstrating? Correct1 Ethnocentrism 2 Multiculturalism 3 Cultural encounter 4 Cultural imposition

1

An older adult resident of a nursing home who has the diagnosis of dementia of the Alzheimer type frequently talks about the good old days at the ranch. What is the most appropriate action by the nurse? Correct1 Allowing the resident to reminisce about the past and listening with interest 2 Involving the resident in interesting diversional activities with a small group 3 Reminding the resident that those "good old days" are past and that the client should focus on the present 4 Introducing the resident to other residents with the same diagnosis so that they can share their past experiences

1

The nurse is caring for an Asian client who had a laparoscopic cholecystectomy six hours ago. When asked whether there is pain, the client smiles and says, "No." What should the nurse do? Correct1 Monitor for nonverbal cues of pain 2 Check the pressure dressing for bleeding 3 Assist the client to ambulate around his room 4 Irrigate the client's nasogastric tube with sterile water

1

The nurse is caring for the client posttranssphenoidal hypophysectomy. When assessing the client, the nurse observes clear drainage from the nares. What could be the cause of this drainage? Correct1 A cerebral spinal fluid leak from an opening to the brain. 2 A normal occurrence for this client's procedure. 3 The client is developing an infection. 4 The client may have had a cold preoperatively, and the nurse will continue to monitor

1

The nurse is interviewing a client admitted for uncontrolled diabetes after binging on alcohol for the past 2 weeks. The client states, "I am worried about how I am going to pay my bills for my family while I am hospitalized." Which statement by the nurse would best elicit information from the client? Correct1 "You are worried about paying your bills?" 2 "Don't worry; your bills will get paid eventually." 3 "When was the last time you were admitted for hyperglycemia?" 4 "You really shouldn't be drinking alcohol because of your diagnosis of diabetes."

1

What is the most important nursing action involved in caring for a client using medications to manage disease? Correct1 Administering the medications 2 Teaching about the medications 3 Ensuring adherence to the medication regimen 4 Evaluating the client's ability to self-administer medications

1

What is the priority nursing intervention in the planning of nursing care for an adolescent client with anorexia nervosa? Correct1 Rewarding weight gain by increasing privileges 2 Discussing the importance of eating a balanced diet 3 Encouraging the client to include high-calorie foods in the diet 4 Family therapy focusing on the influence of the client's behavior on the family

1

A mother brings her 9-month-old infant to the clinic. The nurse is familiar with the mother's culture and knows that belly binding to prevent extrusion of the umbilicus is a common practice. The nurse accepts the mother's cultural beliefs but is concerned for the infant's safety. What variation of belly binding does the nurse discourage? 1 Coin in the umbilicus 2 Tight diaper over the umbilicus 3 Binder that encircles the umbilicus 4 Adhesive tape across the umbilicus

1 Coin in the umbilicus Rationale: A coin may be dislodged, allowing the infant to put it in his or her the mouth, resulting in a safety issue.

A nurse is formulating a teaching plan for a client recently diagnosed with type 2 diabetes. What interventions should the nurse include to decrease the risk of complications? Select all that apply. Correct1 Examine the feet daily Correct2 Wear well-fitting shoes Correct3 Perform regular exercise 4 Powder the feet after showering 5 Visit the primary healthcare provider weekly 6 Test bathwater with the toes before bathing

1,2,3

Which statements describe a mentally healthy person? Select all that apply. Correct1 One who accepts aging Correct2 One who engages available strengths Incorrect3 One who maintains minimum autonomy Correct4 One who sustains positive relationships Incorrect5 One who engages available weaknesses

1,2,4

What are the primary causes of adrenal insufficiency? Select all that apply. Correct1 Hemorrhage Correct2 Tuberculosis 3 Pituitary tumors 4 Postpartum pituitary necrosis Correct5 Acquired immune deficiency syndrome

1,2,5

Which exogenous conditions are responsible for increased cortisol secretion? Select all that apply. Correct1 Asthma 2 Adrenal adenomas Correct3 Cancer chemotherapy Correct4 Organ transplantation 5 Carcinomas of the lung

1,3,4

Which life-threatening wounds are treated with hyperbaric oxygen therapy? Select all that apply. Correct1 Burns 2 Skin cancer Correct3 Osteomyelitis Correct4 Diabetic ulcers 5 Myocardial infarction

1,3,4

A client is admitted to the hospital and benazepril is prescribed for hypertension. Which is an appropriate nursing action for clients taking this medication? 1 Monitor the electroencephalogram (EEG). Correct2 Assess for dizziness. 3 Administer the drug after meals. 4 Assess for dark, tarry stools.

2

A client with a history of atrial fibrillation has a stroke, and vascular dementia (multiinfarct dementia) is diagnosed. In a comparison of assessment findings in clients with vascular dementia and dementia of the Alzheimer type, which factor is unique to vascular dementia? 1 Memory impairment Correct2 Abrupt onset of symptoms 3 Difficulty making decisions 4 Inability to use words to communicate

2

Which hormonal deficiency causes breast atrophy in female clients? 1 Growth hormone Correct2 Luteinizing hormone 3 Thyroid-stimulating hormone 4 Adrenocorticotropic hormone

2

To provide appropriate psychosocial support to clients, a nurse must understand development across the life span. What theory is the nurse using in considering relationships and resulting behaviors as the central factors that influence development? 1 Cognitive theory 2 Psychosocial theory Correct3 Interpersonal theory 4 Psychosexual theory

3

What should a nurse consider about the past experiences of clients who have immigrated to this country? 1 It affects all of their inherited traits. 2 There will be little impact on their lives today. Correct3 It is important that their values be assessed first. 4 How they will interact is permanently established.

3

A registered nurse is explaining the term "just culture" to the student nurse. Which explanation provided by the registered nurse is accurate? 1 "It refers to the agreement to keep promises." 2 "It refers to taking positive actions to help others." 3 "It refers to the ability to answer for one's actions." Correct4 "It refers to promoting open discussion whenever error occurs without fear of recrimination."

4

A client reports to the nurse sleeping until noon every day and taking frequent naps during the rest of the day. What should the nurse do initially?

Arrange a referral for a thorough medical evaluation R: This behavior is a sign of hypersomnia, and the client needs a medical assessment; it is commonly caused by central nervous system damage or certain kidney, liver, or metabolic disorders. Exercise is appropriate for a client experiencing insomnia, not hypersomnia. This behavior is a sign of hypersomnia, and medical causes should be ruled out before attributing it to a psychogenic cause. Narcolepsy consists of recurrent sudden waves of overwhelming sleepiness that occur during the day, even during activities such as eating or conversing.

For which families should the nurse include information related to safe sleeping habits as a priority during a scheduled health maintenance visit for a toddler-age child? Select all that apply.

Asian, African, and Hispanic Americans Asian American, African American, and Hispanic American families are more likely to co-sleep with their toddler-age children; therefore, it is essential for the nurse to teach information that enhances safety with this practice for these families. Irish American and Caucasian American clients are less likely to engage in co-sleeping with their children.

A daughter of a Chinese-speaking client approaches a nurse and asks multiple questions while maintaining direct eye contact. What culturally related concept does the daughter's behavior reflect? Prejudice Stereotyping Assimilation Ethnocentrism

Assimilation Rationale Assimilation involves incorporating the behaviors of a dominant culture. Maintaining eye contact is characteristic of the American or Canadian culture and not of Asian cultures. Prejudice is a negative belief about another person or group and does not characterize this behavior. Stereotyping is the perception that all members of a group are alike. Ethnocentrism is the perception that one's beliefs are better than those of others.

Which intellectual factor would the nurse find appropriate as a dimension for gathering data for a client's health history?

Attention Span R: Attention span is an intellectual dimension used to gather data for a health history. A social dimension for gathering health history includes primary language. A coping mechanism is considered to be a social subdimension used to gather a client's health history data. Physical and developmental subdimensions would include activities and coordination.

Which ethnic group has a greater incidence of osteoporosis due to musculoskeletal differences? Irish Americans African Americans Chinese Americans Egyptian Americans

Chinese Americans Rationale Chinese Americans have an increased incidence of osteoporosis because they have shorter and smaller bones with lower bone density. Irish Americans have taller and broader bones than other Euro-Americans. African Americans have a decreased incidence of osteoporosis. Egyptian Americans are shorter in stature than Euro-Americans and African Americans.

What are the four core roles for the advanced practice registered nurse (RN)? Select all that apply.

Clinical nurse specialist Certified nurse midwife Certified RN anesthetist Certified nurse practitioner

According to the Centers for Disease Control and Prevention, compared to Caucasians, the syphilis rates among Hispanics are two times higher in 2011. What may be the reason for this?

Difference in the status of health literacy One cause of the higher rates of syphilis among Hispanic clients could be a lack of health literacy. Presence of equitable health care support would reduce, not cause a health disparity, as would availability of health care facilities. There is no known genetic predisposition to syphilis among any racial/ethnic group.

The parents of an autistic child begin family therapy with a nurse therapist. The father states that the family members wish to share their religious beliefs with the therapist. What should the nurse do?

Encourage family discussion of their religion in the sessions. If religious beliefs are a family concern, the nurse should allow discussion of the family's thoughts and feelings on the subject; the discussion should be encouraged, not limited. The role of the nurse is to facilitate and listen, not to participate in a mutual discussion about religious beliefs. The religious leader is not part of the family unit and should be invited only if this is requested by the family.

A client who has liver failure says, "I have complete trust in God and I am sure he will take care of my family even if I am not here." Which concept does this most exemplify?

Faith The client's trust that God will take care of his or her family exemplifies faith. An example of religion would be if the client carried out specific rituals or practices to cope. Connectedness involves finding comfort through one's relationship with oneself, other people, and or with a higher power. Transcendence is the belief in a greater force outside of the material world.

Which nursing interventions can help a terminally ill client cope with feelings related to death? Select all that apply.

Helping the client to find meaning and purpose in life by listening to his or her concerns Allowing time for religious readings, spiritual visitations, or attendance at religious services Encouraging the client to pray if he or she wishes by facilitating privacy and a proper environment Feelings of connectedness are important for the client who is terminally ill; therefore, the nurse should promote connectedness by helping the client find meaning and purpose in life by listening to his or her concerns. Prayer and devotion can help the client cope with feelings related to death, so the nurse should allow time for religious readings, spiritual visitations, or attendance at religious services. The nurse can also encourage the client to pray if he or she wishes by facilitating privacy and a proper environment. To help the client to cope with the pain, the nurse should provide medications and therapies for pain management. To help the client manage other aspects of the illness, the nurse can educate the client about complementary medicine.

After changing a dressing that was used to cover a draining wound on a client with vancomycin-resistant enterococci (VRE), the nurse should take which step to ensure proper disposal of the soiled dressing?

Place the dressing in a red bag/hazardous materials bag.

A foreign language-speaking client needs to undergo chemotherapy; a signed consent form is required. What should the nurse do to explain the terms of the consent to the client? Seek the help of an official interpreter. Seek the help of the primary healthcare provider to assist the client. Seek help from the client's family friend who speaks the client's language. Seek help from the client's caregiver who speaks the same language as the client.

Seek the help of an official interpreter. Rationale The nurse should seek the help of an official interpreter to explain the terms of consent to the client. The nurse should not ask for the primary healthcare provider's assistance because he or she might not know the language. The nurse should not seek help from the client's family friend who speaks the language because he or she is not authorized to interpret health information. The nurse should not seek help from the client's caregiver who speaks the same language because he or she should not interpret health information.

Obesity in children is an ever-worsening problem. What concept should a nurse consider when caring for school-aged children who are obese? Enjoyment of specific foods is inherited. There are familial influences on childhood eating habits. Childhood obesity is usually not a predictor of adult obesity. Children with obese parents are destined to become obese themselves.

There are familial influences on childhood eating habits. Rationale Studies have demonstrated that culture and family eating habits have an impact on a child's eating habits. Inheritance is not known to influence eating habits, although it is believed that other hereditary factors are associated with obesity. Childhood obesity is a known predictor of adult obesity. Children with obese parents are not necessarily destined to become obese themselves.

A nurse is caring for an adult client who immigrated to this country 5 years ago. What does the nurse know about the past experiences of clients who have immigrated to this country? They affect their inherited traits. They have little effect on their lives today. They are important in assessment of their values. They establish personal interactions throughout life.

They are important in assessment of their values. Rationale Past experiences are important and must be recognized because they set the parameters for the individual's enduring values throughout life. Past experiences do not affect inherited traits. Past experiences play an important role in an individual's life. Nothing establishes how an individual responds over a lifetime; new experiences continue to influence future responses.

to obtain the stroke volume

the cardiac output should be divided by pulse rate

Regular cow's milk is avoided in infants during the first year of life because it is

too concentrated for an infant's kidneys to manage. It also increases the risk of milk product allergies and is also a poor source of iron and vitamins C and E. Honey and corn syrup are potential sources of botulism toxin. Most infants are not allergic to or intolerant of cow's milk.

A nurse understands that the effects of different variables on a client's health beliefs and practices can help healthcare providers to plan and deliver individualized care. Which statement made by the client should the nurse consider as an influence of the client's intellectual background on his or her health beliefs?

"Don't include eggs in my diet because eggs contribute to excess body heat." If the client states that eggs should not be included in his or her diet because they cause excess body heat, this statement is an example of the influence of the client's intellectual background on his or her health beliefs. If the client states that seafood or ham should not be included in his or her diet because it is against his or her beliefs, this statement is an example of the influence of the client's cultural background on his or her health beliefs. If the client says that he or she does not smoke or drink because these drugs are a major sin, this statement is an example of spiritual factors influencing his or her health beliefs and practices. If the client says that he or she has stopped taking prescribed medications because he or she has recently lost their job, this statement is an example of the client's socioeconomic influence on his or her health beliefs and practices.

The nurse is caring for a client who self-identifies as having a strong Hispanic heritage. According to transcultural nursing researchers, what should the nurse ask when assessing a woman with depression for the risk of self-harm who also identifies as having Hispanic ethnicity?

"How do you express yourself when you're angry?" Transcultural nursing researchers who have studied Hispanic women for the purpose of providing them better nursing care have learned from these women that their culture tends to limit how they can express anger acceptably, which has also shown to result in a higher risk of suicide. The nurse has initiated exploration of potential culture-based issues with this client by asking about her cultural heritage. Asking the client how she expresses anger is the next step the nurse should take to validate (or invalidate) any known cultural characteristics that influence this individual client that may be contributing to her depression. Although questions about seeing friends, having outside interests, or noticing depression are appropriate, they are not as culture-focused as limits on anger expression can be for many women of Hispanic ethnicity.

The registered nurse is preparing to perform a physical assessment of a client with darker skin who is suspected to have jaundice. Which statement by the nurse indicates effective technique?

"I will examine the sclera closest to the iris." The most important assessment for clients who are suspected to have jaundice is examining the sclera closest to the iris. Assessments of the conjunctiva, lips, tongue, nail beds, palms, and soles are effective for a client who is suspected to have cyanosis.

The nurse is working with a female client who is from Southeast Asia that presents with general fatigue. The nurse asks the client if there is any reason for her fatigue. Which statements are most culturally consistent with a client from Southeast Asian culture? Select all that apply.

"I'm tired because I have less blood in my body." "I'm tired because my blood vessels are weak. "I'm tired because I haven't been eating right lately." R: Different cultures may have different explanations for sickness or fatigue. Many Southeast Asian women believe that fatigue occurs because of Ayurveda, or the balance of different energies in the body. This client may believe her fatigue is caused by low blood volume, weak blood vessels, or poor nutrition that has affected the body's balance. A European-American client is more likely to give a biomedical explanation for fatigue such as stress, family issues, or depression.

An African-American woman is diagnosed with primary hypertension. She asks, "Is hypertension a disease of African-American people?" What is the nurse's best response?

"The higher-risk population is composed of African-American men and women." African-Americans represent a higher-risk population than Caucasian-Americans for hypertension; the reason is unknown. African-American women are more frequently affected by hypertension than are Caucasian women. African-Americans of both sexes have a higher prevalence than Caucasian-Americans of both sexes. African-American women have a higher risk than African-American men.

An African-American woman is diagnosed with primary hypertension. She asks, "Is hypertension a disease of African-American people?" What is the nurse's best response? "The prevalence of hypertension is about equal for women of all races." "The higher-risk population is composed of African-American men and women." "The highest-risk population consists of older Caucasian-American men and women." "The prevalence of hypertension is greater for African-American men than for African-American women."

"The higher-risk population is composed of African-American men and women." Rationale African-Americans represent a higher-risk population than Caucasian-Americans for hypertension; the reason is unknown. African-American women are more frequently affected by hypertension than are Caucasian women. African-Americans of both sexes have a higher prevalence than Caucasian-Americans of both sexes. African-American women have a higher risk than African-American men.

A registered nurse is teaching a nursing student about the importance of values in nursing practice. Which information provided by the registered nurse is appropriate? Select all that apply.

"Values vary among clients and develop and change over time." "The values that an individual holds reflect cultural and social influences." "To discuss differences in opinions and values, the nurse should be clear about his or her own values." The nurse should know that values vary among people and develop and change over time. The nurse should know that the values an individual hold reflect cultural and social influences. The nurse should know that it is important to be clear about one's own values before discussing the differences of opinions and values. People consider strong values as facts rather than opinions. The nurse should never evaluate the client's values and beliefs in terms of his or her own values and beliefs.

The nurse is teaching the Hispanic parents of a preschool child about the prevention of lead poisoning. Which statement by the parents indicates a need for further teaching?

"We can use orange powders for diarrhea." Greta and azarcon (also known as alarcon, coral, luiga, maria luisa, and rueda), traditional Hispanic remedies taken for upset stomach, constipation, diarrhea, and vomiting, are also used for teething babies. Both are fine orange powders with a lead content as high as 90%. Further teaching is required if the family indicates that they will continue treating diarrhea with a home remedy. Food should not be stored in open cans, particularly those that have been imported. Cold water for consumption (drinking, cooking, and especially reconstitution of powdered infant formula) should be used; hot water dissolves lead more quickly than does cold water, yielding a higher level of lead. Frequent healthy snacks are encouraged because lead is absorbed better on an empty stomach.

One morning a client with the diagnosis of acute depression says, "God is punishing me for my past sins." What is the best response by the nurse?

"You sound very upset about this." The response focuses on the client's feelings rather than the statement, and it serves to open channels of communication. "Why do you think that?" asks the client to decide what is causing the feelings; most people are unable to explain why they feel as they do. "Do you believe that God is punishing you for your sins?" simply echoes the client's statement and does not reflect feelings or stimulate further communication. "If you feel this way, you should talk to your spiritual advisor" does nothing to stimulate further communication; in fact, it tells the client to talk about the feelings with someone else.

A registered nurse educates a student nurse regarding the appropriate method of dealing with clients of different cultural backgrounds. Which statements by the student nurse indicate an understanding of various cultures? Select all that apply.

- "I should be aware of my own cultural background and beliefs when attending to clients who belong to different cultures." - "I should focus on understanding the traditions, beliefs, and values of the client's culture."

A nurse understands that the effects of different variables on a client's health beliefs and practices can help healthcare providers to plan and deliver individualized care. Which statements made by the client should the nurse consider as an influence of spirituality on health beliefs? Select all that apply. 1 "My faith prohibits the use of a donor's sperm." 2 "Don't administer nasal drops now because it will break my fast." 3 "I do not believe in surrogacy because this is not permitted in our community." 4 "I am not worried about my surgery because I have undergone several surgeries in the past." 5 "I am not able to meet my basic needs. How do you expect me to buy these costly medications?"

-1 "My faith prohibits the use of a donor's sperm." -2 "Don't administer nasal drops now because it will break my fast." -3 "I do not believe in surrogacy because this is not permitted in our community."

Which nursing interventions are examples of the nurse as a caregiver? Select all that apply. 1 Encouraging the client to exercise daily 2 Setting goals for the client to reduce weight 3 Arranging for the client to meet a spiritual adviser 4 Evaluating the client's understanding of prescribed diet 5 Demonstrating the procedure to self-administer insulin injection

-1 Encouraging the client to exercise daily -2 Setting goals for the client to reduce weight -3 Arranging for the client to meet a spiritual adviser

An elderly adult suffered an injury after falling down in the washroom. The primary healthcare provider performed a surgical procedure on the client and orders a blood transfusion. A family member of the client mentions that blood transfusions are not permitted in their community. What should the nurse do in order to handle the situation? 1 The nurse should wait for the court's order to give blood to the client. 2 The nurse should proceed with the transfusion in order to save the client's life. 3 The nurse should inform the primary healthcare provider and not give blood to the client. 4 The nurse should explain to the family member that the client needs this transfusion.

-3 The nurse should inform the primary healthcare provider and not give blood to the client.

A 5-year-old child who is newly arrived from Latin America attends a nursery school where everyone speaks English. The child's mother tells the nurse that her child is no longer outgoing and has become very passive in the classroom. What is the probable reason for the child's behavior? Correct1 Culture shock 2 Social immaturity 3 Experience of discrimination 4 Lack of interest in school activitie

1

After flushing a client's left forearm saline lock (SL) with normal saline, the client begins to report a painful and burning sensation at the insertion site. Which is the most appropriate action for the nurse to take? Correct1 Remove the angiocatheter and saline lock and restart the SL in another site. 2 Document the findings per protocol and reassess the site in eight hours. 3 Flush the angiocatheter and saline lock again with sterile water. 4 Change the dressing and apply a new clean dressing.

1

An arterial blood gas report indicates the client's pH is 7.25, PCO2 is 35 mm Hg, and HCO3 is 20 mEq/L. Which disturbance should the nurse identify based on these results? Correct1 Metabolic acidosis 2 Metabolic alkalosis 3 Respiratory acidosis 4 Respiratory alkalosis

1

An older depressed person at an independent living facility constantly complains about her health problems to anyone who will listen. One day the client says, "I'm not going to any more activities. All these old crabby people do is talk about their problems." What defense mechanism does the nurse conclude that the client is using? Correct1 Projection 2 Introjection 3 Somatization 4 Rationalization

1

Electroconvulsive therapy (ECT) is a mode of treatment that is used primarily to treat what? Correct1 Clinical depression 2 Substance abuse disorders 3 Antisocial personality disorder Incorrect4 Psychosis occurring in schizophrenia

1

A resident in a nursing home recently immigrated to the United States from Italy. How does the nurse plan to provide emotional support? 1 By offering choices consistent with the client's heritage 2 By assisting the client in adjusting to American culture 3 By ensuring that the client understands American beliefs 4 By correcting the client's misconceptions about appropriate health practices

1 By offering choices consistent with the client's heritage

A public health nurse is working with a family with three school-aged children as the unit of service. What should the nurse consider when caring for this family? 1 Certain members of the family may be capable of giving more support than the nurse. 2 Assessing each family member is not necessary to plan care for the family as a whole. 3 Family values are not as important as other factors regarding how assistance is perceived. 4 Helping the family requires separating health problems from other aspects of the family's life.

1 Certain members of the family may be capable of giving more support than the nurse.

What criteria should the nurse use to determine normal sinus rhythm for a client on a cardiac monitor? Select all that apply. Correct1 The RR intervals are relatively consistent. Correct2 One P wave precedes each QRS complex. 3 The ST segment is higher than the PR interval. 4 Four to eight complexes occur in a 6-second strip. 5 The QRS complex ranges from 0.12 to 0.2 seconds.

1,2

Which nursing assessment questions assess the faith, belief, fellowship, and community aspect of a client's spirituality? Select all that apply.

1. "What gives meaning to your life?" 2. "What is your source of power, hope, and belief during difficult times?" 3. "In what way do your beliefs help or strengthen you for coping with illness?" A nurse can assess the faith, belief, fellowship, and community aspect of a client's spirituality by asking a client what gives meaning to his or her life, about his or her source of power, hope, or belief during difficult times, and about how his or her beliefs help or strengthen him or her for coping with illness. When a nurse asks the client in what way illness affects his or her capability to express what is essential in life, it helps in assessing the vocation aspect of spirituality. When a nurse asks the client how he or she feels about the changes that have been caused by the illness, it helps in assessing the life and self-responsibility aspect of spirituality.

Which behavior is seen in children at the undifferentiated stage of spiritual development, as propounded by Fowler? 1Children have no concept of right or wrong to guide their behaviors. 2Children imitate the religious behaviors without comprehending any meaning. 3Children reason and question some of the established parental religious standards. 4Children have a reverence for religious matters and are able to articulate their faith

1Children have no concept of right or wrong to guide their behaviors

An elderly client states, "Disease occurs when supernatural elements enter the body." Which variable influences the client's health beliefs in this scenario? 1Spiritual factors 2Emotional factors 3Intellectual background 4Perception of functioning

1Spiritual factors

A depressed, withdrawn client exhibits sadness through nonverbal behavior. What should the nurse plan to help the client to do? 1 Increase structured physical activity. Correct2 Cope with painful feelings by sharing them. 3 Decide which unit activities can be performed. 4 Improve ability to communicate with significant others.

2

A hyperactive, acting-out 9-year-old boy is started on a behavior modification program in which tokens are given for acceptable behavior. When he begins to lose a game he is playing with other children, he begins to kick the other children under the table and call them names. What is the most appropriate behavior modification technique for the nurse to use? 1 Ignoring the child's behavior Correct2 Placing the child in a time-out 3 Taking the child's daily allotment of tokens away 4 Engaging the child in a conversation about good sportsmanship

2

A nurse is teaching parents of toddlers about why children receiving specific medications should not receive varicella vaccines. Which medication will be included in the discussion? 1 Insulin Correct2 Steroids Incorrect3 Antibiotics 4 Anticonvulsants

2

Nurses on a psychiatric unit have secluded a client who has the diagnosis of bipolar I disorder, manic episode, and who has been losing control and throwing objects while in the dayroom. What is the most important intervention for the client who is given an as-needed (PRN) medication and confined to involuntary seclusion? 1 Continue intensive nursing interactions. Correct2 Evaluate the client's progress toward self-control. 3 Determine whether any staff member has been injured. Incorrect4 Observe the client for side effects of the medication given to the client.

2

Which statement related to breast-feeding is correct? Incorrect1 Protein soluble drugs can enter breast milk. Correct2 Drugs with a long half-life should be avoided. 3 Mothers should take drugs prior to breast-feeding. 4 Drug usage during lactation is safe because the drugs will not harm the baby

2

A nurse understands that the effects of different variables on a client's health beliefs and practices can help healthcare providers to plan and deliver individualized care. Which statement made by the client should the nurse consider as an influence of the client's intellectual background on his or her health beliefs? 1"Don't include seafood or ham in my diet because it is against my beliefs." 2 "Don't include eggs in my diet because eggs contribute to excess body heat." 3"I do not smoke or drink because these intoxicants are major sins." 4"I am not taking any of my prescribed medications because I recently lost my job."

2 "Don't include eggs in my diet because eggs contribute to excess body heat."

How should nurses provide effective nursing care to clients from different cultural backgrounds? 1 By advising clients that some cultural practices may be harmful to health 2 By providing care that fits the clients' cultural beliefs 3 By strictly adhering to organization policies regarding nursing care 4 By ignoring the cultural aspect and focusing on the medical aspect of care

2 By providing care that fits the clients' cultural beliefs

A pregnant client states, "Abortion is banned in our community because it interferes with God's creative work." According to the nurse, which variable influences the client's health belief? 1 Emotional factors 2 Cultural background 3 Socioeconomic factors 4 Perception of functioning

2 Cultural background

A client proclaims that he is "the second son of God." What type of delusion does the nurse identify? 1 Influence 2 Religious 3 Reference 4 Persecutory

2 Religious

One morning a client with the diagnosis of acute depression says, "God is punishing me for my past sins." What is the best response by the nurse? 1"Why do you think that?" 2"You sound very upset about this." 3"Do you believe that God is punishing you for your sins?" 4"If you feel this way, you should talk to your spiritual adviser."

2"You sound very upset about this."

An adolescent female suffering from severe cystic acne is placed on isotretinoin. What important facts should the nurse tell the client about isotretinoin? Select all that apply. 1 Inform the client to use vinyl helmet straps. Correct2 Inform the client about the risk of teratogenicity. Correct3 Inform the client that skin improvement may take time. 4 Inform the client to scrub vigorously to remove blackheads. 5 Inform the client to use abrasive cleansers to remove blackheads.

2,3

Which questions will allow the nurse to assess a preschool-age child diagnosed with asthma for delayed peer relationships? Select all that apply. 1 "Can your child independently dress each day?" Correct2 "Does your child use 'baby-like' terms when talking?" Correct3 "Does your child play with the other children in the playroom?" 4 "Has your child ever thought that the asthma is a punishment?" 5 "Does your child become anxious before respiratory treatments?"

2,3

A nurse is managing the care of a client with recently diagnosed schizophrenia. Effective therapeutic communication will directly affect which client-focused outcomes? Select all that apply. 1 The client will become capable of part-time employment. Correct2 The client will effectively express emotional and physical needs. 3 The client will demonstrate wellness reflective of physical potential. Correct4 The client will demonstrate an understanding of the mental health disorder. Correct5 The client will recognize the issues most important to managing this disorder.

2,4,5

A client has been diagnosed with generalized anxiety disorder (GAD). Which behavior supports this diagnosis? 1 Making huge efforts to avoid "any kind of bug or spider" 2 Experiencing flashbacks to an event that involved a sexual attack Correct3 Spending hours each day worrying about something "bad happening" 4 Becoming suddenly tachycardic and diaphoretic for no apparent reason

3

A client is admitted to the psychiatric hospital after many self-inflicted nonlethal injuries over the preceding month. Of which level of suicidal behavior is the client's behavior reflective? 1 Threats 2 Ideation Correct3 Gestures 4 Attempts

3

A client is presented with the treatment option of electroconvulsive therapy (ECT). After discussion with staff members, the client requests that a family member be called to help make the decision about this treatment. What ethical principle does the nurse consider when supporting the client's request? 1 Justice 2 Veracity Correct3 Autonomy 4 Beneficence

3

A client is receiving total parenteral nutrition. The nurse assesses for which client response that indicates hyperglycemia? 1 Paralytic ileus 2 Respiratory rate below 16 Correct3 A fruity odor to the breath 4 Serum glucose of 105 mg/100 mL

3

A client who had a femoropopliteal bypass graft is receiving clopidogrel postoperatively. What should the nurse teach the client related to the medication? 1 Eliminate grapefruit from the diet 2 Eat more roughage if constipation occurs Correct3 Report any occurrence of multiple bruises 4 Take the medication on an empty stomach

3

A nurse is speaking with a client who was sexually abused as a child. The client does not know what constitutes inappropriate touch by another person. What issue will have to be addressed with this client? 1 Increased libido 2 Phobic behavior Correct3 Boundary violations 4 Excessive aggression

3

During a group therapy session some members accuse another client of intellectualizing to avoid discussing feelings. The client asks whether the nurse agrees with the others. What is the best response by the nurse? 1 "It seems that way to me, too." 2 "What's your perception of my behavior?" Correct3 "Are you uncomfortable with what you were told?" 4 "I'd rather not give my personal opinion at this time."

3

During data collection, the nurse inspects the client's nose and concludes that the client has an infection. Which finding supports the nurse's conclusion? 1 Bloody discharge 2 Watery discharge Correct3 Thick mucosal discharge Incorrect4 Purulent and malodorous discharge

3

Three days after a stressful incident a client can no longer remember why it was stressful. The nurse, in relating to this client, can be most therapeutic by identifying that the inability to recall the situation is an example of what defense mechanism? 1 Denial 2 Regression Correct3 Repression 4 Dissociation

3

When talking with one of the day nurses, a client with the diagnosis of anorexia nervosa states that the day nurses give better care and are nicer than the night nurses. The client also asks a question that the day nurse knows was already answered by one of the night nurses. What conclusion should the nurse make about the client? 1 The client needs assistance in exploring and verbalizing feelings about the night nurses. 2 The client is trying to develop a bond of trust with a staff member that should be supported. Correct3 The client is trying to divide the staff, and the behavior should be reported to the other staff members. 4 The client has negative feelings about the night nurses, and the nurses should be informed of these feelings.

3

Which characteristic does the nurse associate with a punch biopsy? 1 It is usually indicated for superficial or raised lesions. 2 It is more uncomfortable than other biopsies while healing. Correct3 It is performed using a circular cutting instrument 2 to 6 mm in diameter. 4 It removes only the portion of the skin that rises above the surrounding tissue.

3

Which nursing intervention is most important for a client who has the diagnosis of antisocial personality disorder? 1 Teaching and modeling assertiveness 2 Using a gentle and reassuring approach Correct3 Providing clear boundaries and consequences 4 Presenting an empathetic and democratic approach

3

The mother of a preschool-age child tells the school nurse that her husband is dying of cancer and that she is worried about how her child will cope. As part of their discussion, what does the school nurse include that preschool-age children view death as? 1 Universal 2 Irreversible 3 A form of sleep 4 A frightening ghost

3 A form of sleep

A 15-year-old emancipated minor gave birth to a boy 36 hours ago and has requested a circumcision. What is the medical priority? 1Obtaining a physician's prescription for a lidocaine injection 2Educating the new mother about the circumcision procedure 3Obtaining an informed consent signed by the mother of the baby 4Obtaining an informed consent signed by the grandmother of the baby

3Obtaining an informed consent signed by the mother of the baby

A client has a "prayer cloth" pinned to the hospital gown. The cloth is soiled from being touched frequently. What should the nurse do when changing the client's gown? 1Make a new prayer cloth. 2Discard the soiled prayer cloth. 3Pin the prayer cloth to the clean gown. 4Wash the prayer cloth with a mild detergent

3Pin the prayer cloth to the clean gown.

A client in the mental health clinic who has concerns about getting married says to the nurse, "I guess I'd better get married. All the plans are made and paid for, and the invitations have all been mailed." What defense mechanism is the client using? 1 Introjection 2 Identification 3 Compensation Correct4 Rationalization

4

A client is admitted to the hospital for replacement of the mitral valve. The primary purpose of the nurse checking the pulses in the client's legs frequently after surgery is detection of what? 1 Atrial fibrillation 2 Postsurgical bleeding Incorrect3 Arteriovenous shunting Correct4 Peripheral thrombophlebitis

4

A client was recently given a diagnosis of a manic episode of bipolar I disorder. What activity is most therapeutic for this client at this time? 1 Doing a craft project 2 Playing a game of table tennis 3 Playing cards with another patient Correct4 Walking around the unit with a nurse

4

A client who recently was diagnosed as having myelocytic leukemia discusses the diagnosis by referring to statistics, facts, and figures. The nurse determines that the client is using which defense mechanism? 1 Projection 2 Sublimation 3 Identification Correct4 Intellectualization

4

A client with cellulitis of the leg asks why bed rest has been prescribed to prevent sepsis. Which purpose will the nurse explain to the client? 1 This decreases catabolism to promote healing at the site of injury. 2 This lowers the metabolic rate in an attempt to help reduce the fever. 3 This reduces the energy demands on the body in the presence of infection. Correct4 This limits muscle contractions that may force causative organisms into the bloodstream.

4

A client with schizophrenia is speaking made-up words that have no meaning to other people. What term should the nurse use to document these verbalizations? 1 Avolition 2 Echolalia 3 Anhedonia Correct4 Neologisms

4

A client with the diagnosis of schizophrenia refuses to eat meals. Which nursing action is most beneficial for this client? 1 Directing the client repeatedly to eat the food 2 Explaining to the client the importance of eating 3 Waiting and allowing the client to eat whenever the client is ready Correct4 Having a staff member sit with the client in a quiet area during mealtimes

4

A nurse is caring for a client with Addison's disease. Upon assessment, which classic sign will the nurse find? 1 Ecchymosis 2 Hyperreflexia 3 Exophthalmos Correct4 Hyperpigmentation

4

A nurse is caring for a client with vascular dementia. What does the nurse expect of this client's mental status? 1 Diminished remote memory resulting from anoxia 2 Loss of abstract thinking related to emotional state 3 Inability to concentrate related to decreased stimuli Correct4 Difficulty recalling recent events related to cerebral hypoxia

4

A nurse is caring for a newly admitted client with obsessive-compulsive disorder. When should the nurse anticipate that the client's anxiety level will increase? 1 As the day progresses 2 When family members visit 3 During a physical assessment by the nurse Correct4 When limits are set on the performance of a ritual

4

A nurse teaches a client about how to protect a skin area that has undergone radiation treatment. Which statement made by the client indicates the nurse needs to follow up? 1 "I should avoid swimming in saltwater." 2 "I should avoid using adhesive bandages." 3 "I should avoid wearing tight-fitting cloth." Correct4 "I should avoid rinsing the area with the saline solution."

4

A woman with bipolar disorder, manic episode, has been spending thousands of dollars on clothing and makeup. She has been partying in bars every night and rarely sleeps or eats. The nurse in the outpatient clinic, knowing that this client rarely eats, recognizes that her eating problems most likely result from what? 1 Feelings of guilt 2 Need to control others 3 Desire for punishment Correct4 Excessive physical activity

4

After determining that the nurses on the psychiatric unit are uncomfortable caring for clients who are from different cultures than their own, the nurse manager establishes a unit goal that by the next annual review the unit will have achieved what? 1 Increased cultural sensitivity 2 Decreased cultural imposition 3 Decreased cultural dissonance Correct4 Increased cultural competence

4

An adolescent client with antisocial personality disorder plans to live with the parents after discharge. The parents request advice on how to respond to their child's unruly behavior. What is the most therapeutic response by the nurse? 1 "Discuss the behavior with your child and encourage the development of self-control." 2 "Avoid setting expectations for behavior and react to each situation as it arises." 3 "Help your child find new friends and encourage finding a job and assuming personal responsibility." Correct4 "Set clear limits, explain the consequences if your child disregards them, and firmly and consistently apply them."

4

Obesity in children is an ever-increasing problem. What should a nurse consider before confronting the problem with individual children? 1 Enjoyment of specific foods is inherited. 2 Childhood obesity is not usually a predictor of adult obesity. 3 Children with obese parents and siblings are destined for obesity. Correct4 Familial and cultural influences are deciding factors in eating habits.

4

The nurse determines that the plan for bolstering an overweight adolescent's self-esteem has been effective when, 3 months later, the adolescent's mother reports that the adolescent is doing what? 1 Seems to be doing average work in school 2 Has asked her how to bake bread and cookies 3 Imitates a sibling's manner of speech and dress Correct4 Joined a dirt bike group that meets at the school

4

The nurse is providing education to a client with calculi in the calyces of the right kidney. The client is scheduled to have the calculi removed. Which information should the nurse include in the teaching? 1 The surgery will be performed transurethrally. 2 During the surgery, the right ureter will be removed. 3 After surgery, a suprapubic catheter will be in place. Correct4 After surgery, there will be a small incision in the right flank area.

4

The nurse leader is teaching the staff that the health care provider continuously strives to work effectively within the cultural context of a client. Which cultural principle is the nurse leader explaining? 1 Cultural diversity 2 Cultural sensitivity 3 Cultural imposition Correct4 Cultural competence

4

The parents of a preschooler tell the nurse that they try to inculcate good eating habits by asking the child to be at the table until the "plate is clean." What condition is the child at risk for? 1 Anorexia 2 Depression 3 Aggression Correct4 Poor eating habits

4

What does a nurse consider the most significant influence on many clients' perception of pain when interpreting findings from a pain assessment? 1 Age and sex 2 Physical and physiological status 3 Intelligence and economic status Correct4 Previous experience and cultural values

4

What is the best drug of choice for treating obsessive-compulsive disorder? 1 Imipramine 2 Lithium salts 3 Amitriptyline Correct4 Clomipramine

4

What nursing intervention will be most effective in helping relieve the anxiety of a young school-aged child during the postoperative period? 1 Encouraging the child to talk about feelings 2 Having the child and a parent room together 3 Telling the child a story about a child with similar surgery Correct4 Providing the child with sterile dressing equipment and a doll

4

Which client has a primary lesion? Incorrect1 One with scales 2 One with ulcers 3 One with fissures Correct4 One with erosions

4

While caring for a client with an intravenous cannula, the nurse assesses the site and finds that it red, swollen, and warm with purulent drainage near the insertion site. Which nursing intervention provides client comfort? 1 Slowing the infusion rate temporarily 2 Elevating the extremity slightly above level 3 Applying cold and warm compresses frequently Correct4 Cleaning the site with alcohol by expressing the drainage

4

While receiving betamimetic (tocolytic) therapy for preterm labor the client begins to experience muscle tremors and exhibit signs of nervousness. She reports, "My heart is racing." The nurse identifies that the client's pulse rate is 110 beats/min and regular. What should the nurse do next? 1 Discontinue the medication as per protocol. 2 Notify the primary healthcare provider that preterm labor has restarted. 3 Obtain the client's laboratory results for electrolyte levels. Correct4 Reassure the client that these are expected side effects of the medication.

4

A nurse is with the parents of a 3-year-old child who has just died. What is the most therapeutic question for the nurse to ask the parents? 1 "Do you feel ready to consent to an autopsy?" 2 "Have you made a decision about organ donation?" 3 "Would you like to talk about how you'll tell your other children?" 4 "Can I be of any help with traditional practices that are important to you?"

4 "Can I be of any help with traditional practices that are important to you?"

A nurse is assessing the grief response of a family member whose relative has died. What must the nurse consider first about the family to conduct an effective assessment? 1 Personality traits 2 Educational level 3 Socioeconomic class 4 Cultural background

4 Cultural background

After determining that the nurses on the psychiatric unit are uncomfortable caring for clients who are from different cultures than their own, the nurse manager establishes a unit goal that by the next annual review the unit will have achieved what? 1Increased cultural sensitivity 2Decreased cultural imposition 3Decreased cultural dissonance 4Increased cultural competence

4 Increased cultural competence

A client has just been admitted to the psychiatric unit on involuntary admission status. During the admission assessment the client tells the nurse, "I am the second son of God and need to say a prayer." What is the best response by the nurse? 1 Interrupting the client and continuing the assessment 2 Joining the client in the prayer and then refocusing on the assessment 3 Quietly leaving the client and coming back later to complete the assessment 4 Waiting until the client finishes the prayer and then completing the assessment

4 Waiting until the client finishes the prayer and then completing the assessment

Before effectively responding to a sexually abused victim on the phone, it is essential that the nurse in the rape crisis center do what? 1Get the client's full name and address. 2Call for assistance from the psychiatrist. 3Know some myths and facts about sexual assault. 4Be aware of any personal bias about sexual assault.

4Be aware of any personal bias about sexual assault.

A nurse manager works on a unit where the nursing staff members are uncomfortable taking care of clients from cultures that are different from their own. How should the nurse manager address this situation? 1Assign articles about various cultures so that they can become more knowledgeable. 2Relocate the nurses to units where they will not have to care for clients from a variety of cultures. 3Rotate the nurses' assignments so they have an equal opportunity to care for clients from other cultures. 4Plan a workshop that offers opportunities to learn about the cultures they might encounter while at work.

4Plan a workshop that offers opportunities to learn about the cultures they might encounter while at work.

When assessing the development of a school-age child, the nurse concludes that the child has normal development according to Fowler's spiritual development. Which behavior helped the nurse reach this conclusion? 1The child imitates the religious gestures of elders. 2The child does not differentiate between right and wrong actions. 3The child has spiritual disappointment and modifies religious practices. 4The child believes God will punish bad behavior and reward good behavior.

4The child believes God will punish bad behavior and reward good behavior.

During a routine checkup a client reports concerns over weight gain despite trying juice cleanses and other trend diets. The nurse records the client's weight and BMI at a healthy range, but the client states, "I wish I were as thin as my co-workers." The client is at risk for what culturally-bound condition? Neurasthenia Anorexia nervosa Shenjing shuairuo Ataque de nervios

Anorexia nervosa Rationale Anorexia nervosa is a Western culture-bound eating disorder characterized by obsession with body image. A client who continues to follow weight loss diets despite being a healthy weight may be at risk for malnutrition. The client with neurasthenia may feel a lack of energy but not necessarily from following a strict diet to maintain body image. Shenjing shuairuo is a condition associated with Chinese culture that focuses on a weakness of nerves and is not associated with eating disorders or body image. Ataque de nervios is a Latino-Caribbean culture-bound syndrome and is not associated with body image.

A client says, "None of the medications will work on me because I am away from my holy land." What course of action should the nurse take to comply with teamwork and collaboration competency according to the Quality and Safety Education for Nurses (QSEN)?

Approach the agency chaplain to discuss the spiritual needs of the client According to Quality and Safety Education for Nurses (QSEN) competency, the nurse complies with teamwork and collaboration competency to function effectively within the nursing and interprofessional teams. In the given scenario, the nurse should collaborate with the agency chaplain to discuss the client's spiritual needs. The nurse complies with the patient-centered care competency by providing care to the client with respect to his or her diversity, values, and beliefs. The nurse complies with the evidence-based practice competency by conducting thorough research on the effect of emotional distress on the client's health. The nurse complies with the quality improvement competency by using the flow chart data to provide the best possible care and monitor the outcome of care processes.

A nurse understands that value clarification is a technique useful in therapeutic communication because initially it helps clients do what? Become aware of their personal values Gain information related to their needs Make correct decisions related to their health Alter their value systems to make them more socially acceptable

Become aware of their personal values Rationale Value clarification is a technique that reveals individuals' values so the individuals become more aware of them and their effect on others. Gaining information, making correct health decisions, and altering value systems to make them more socially acceptable are not outcomes of value clarification.

A client who only speaks Spanish is being cared for at a hospital in which nursing personnel only speak English. What communication technique would be appropriate for the nurse to use when discussing healthcare decisions with the client? Contact an interpreter provided by the hospital. Contact the client's family member to translate for the client. Communicate with the client using Spanish phrases the nurse learned in a college course. Communicate with the client with the use of a hospital-approved Spanish dictionary.

Contact an interpreter provided by the hospital. Rationale Interpreters provided by the healthcare organization should be used to communicate with clients with limited English proficiency to ensure accuracy of communicated information. In hospital settings, it is not suitable for family members to translate healthcare information, but they can assist with ongoing interactions during the client's care. The other options do not ensure accurate interpretation of language.

A nurse is caring for a client who believes that surgery contaminates the soul unless proper cleansing rituals are performed. Which nursing activity adheres to the Quality and Safety Education for Nurses (QSEN) competency patient-centered care?

Coordinating with appropriate spiritual practitioners to help perform the cleansing ritual before the client's surgery Patient-centered care requires the nurse to understand that the client is the source of control when providing care. The nurse should therefore observe and respect any of the client's values, needs, and preferences when delivering health care. The nurse in the given situation complies with the client's wishes by coordinating with appropriate spiritual practitioners to help perform the cleansing ritual before the surgery. Quality improvement requires the nurse to use data for monitoring the outcomes of care processes. The nurse adheres to this competency by evaluating the effects of complying with the requests of clients regarding their psychological health. In order to adhere to the QSEN competency called safety, the nurse should minimize the risk of harm by improving his or her professional performance. The nurse improves his or her communication skills in order to reduce the risk of harming the client emotionally. Evidence-based practice requires nurses to integrate best professional practices with the client's preferences when delivering health care. The nurse complies with this competency by conducting research about the ritualistic practices of the client's community to formulate a care plan.

During a survey, the community nurse meets a client who has not visited a gynecologist after the birth of her second child. The client says that her mother or sister never had annual gynecologic examinations. Which factor is influencing the client's health practice? Spiritual belief Family practices Emotional factors Cultural background

Family practices Rationale Family practices influence the client's perception of the seriousness of diseases. The client does not feel the need to seek preventive care measures because no family member practices preventive care. The client is not influenced by spiritual beliefs in this instance. An individual's spiritual beliefs and religious practices may restrict the use of certain forms of medical treatment. Emotional factors such as stress, depression, or fear may influence an individual's health practice; however, this client does not show signs of being affected by emotional factors. The client is said to be influenced by cultural background if he or she follows certain beliefs about the causes of illness and uses customary practices to restore health.

What rules of impression management should the nurse follow when caring for an Asian client? Select all that apply.

Greet the client and family in their language. Observe the distance maintained by the client. Clarify whether the client wants someone from the family to be present. While caring for a client, the nurse should observe and maintain the distance displayed by the client. In Asian cultures, touching someone is considered disrespectful or unethical. Before examining the client, the nurse should clarify whether the client wants someone from the family to be present. The nurse should greet the client and family in his or her language, if possible. When relatives visit the client, ask for introductions and the visitors' relationships to the client. The nurse should avoid talking to the client in the presence of relatives and friends. In such cases, the nurse should tell them to wait in the waiting room, then talk to the client in private.

After determining that the nurses on the psychiatric unit are uncomfortable caring for clients who are from different cultures than their own, the nurse manager establishes a unit goal that by the next annual review the unit will have achieved what? Increased cultural sensitivity Decreased cultural imposition Decreased cultural dissonance Increased cultural competence

Increased cultural competence Rationale Cultural competence encompasses sensitivity as well as knowledge, desire, and skill in caring for those who are different from one's self. The nurses are already somewhat sensitive to those from different cultures and now must move forward in their ability to care for these clients. The nurses are not imposing their culture on the clients; they are avoiding them. There is no clashing of cultures in this situation.

A client dies during surgery, and the family members ask that the hospital not conduct autopsy examinations. Which religion might the nurse expect this family to practice?

Islam This family is likely to be Muslim; Islam does not allow autopsies. Hinduism, Buddhism, and Christianity allow such postmortem examinations when required.

What should a nurse consider about the past experiences of clients who have immigrated to this country? It affects all of their inherited traits. There will be little impact on their lives today. It is important that their values be assessed first. How they will interact is permanently established.

It is important that their values be assessed first. Rationale Past experiences are important and must be recognized because they help set the individual's values throughout life. Past experiences will not affect inherited traits. Past experiences play an important role in an individual's life. Nothing establishes how an individual responds forever; new experiences continue to influence future responses.

The nurse is caring for an African American client with renal failure. The client states that the illness is a punishment for sins. Which cultural health belief does the client communicate? Yin/Yang balance Biomedical belief Determinism belief Magicoreligious belief

Magicoreligious belief Rationale An African American client may have magicoreligious beliefs, which focuses on hexes or supernatural forces that cause illness. Such clients may believe that illness is a punishment for sins. The yin/yang belief system does not consider illness as a punishment. The biomedical belief system maintains that health and illness are related to physical and biochemical processes with disease being a breakdown of the processes. The belief of determinism focuses on outcomes that are externally preordained and cannot be changed.

The nurse is caring for an Asian client who had a laparoscopic cholecystectomy six hours ago. When asked whether there is pain, the client smiles and says, "No." What should the nurse do? Monitor for nonverbal cues of pain Check the pressure dressing for bleeding Assist the client to ambulate around his room Irrigate the client's nasogastric tube with sterile water

Monitor for nonverbal cues of pain Rationale Asian clients tend to be stoic regarding pain and usually do not acknowledge pain; therefore, the nurse should assess these clients further. This type of surgery does not require pressure dressings. First, the client must be assessed further for pain. If there is pain, the client should ambulate after, not before, receiving pain medication. Postoperatively, nasogastric tubes are irrigated when needed, not routinely.

A client has a "prayer cloth" pinned to the hospital gown. The cloth is soiled from being touched frequently. What should the nurse do when changing the client's gown?

Pin the prayer cloth to the clean gown. The prayer cloth has religious significance for the client and should be preserved as is. Making a new prayer cloth disregards what the prayer cloth means to the client. The prayer cloth is the property of the client and should not be discarded. Washing the prayer cloth with a detergent disregards what the prayer cloth means to the client; this never should be done without the client's permission.

Alternative therapy measures have become increasingly accepted within the past decade, especially in the relief of pain. Which methods qualify as alternative therapies for pain? Select all that apply.

Prayer 2 Hypnosis 4 Aromatherapy 5 Guided imagery Prayer is an alternative therapy that may relax the client and provide strength, solace, or acceptance. The relief of pain through hypnosis is based on suggestion; also, it focuses attention away from the pain. Some clients learn to hypnotize themselves. Aromatherapy can help relax and distract the individual and thus increase tolerance for pain, as well as relieve pain. Guided imagery can help relax and distract the individual and thus increase tolerance for pain, as well as relieve pain. Analgesics, both opioid and nonopioid, long have been part of the standard medical regimen for pain relief, so they are not considered an alternative therapy.

A client with a terminal illness is grateful for the care received in the hospital and has slowly started to come to terms with imminent death. The nurse recognizes that the client's behavior and attitude is most consistent with which cultural group?

Somalian Culture Terminally ill clients who belong to the Somalian culture may slowly accept their imminent death and have faith in God. Somalian clients will generally express their gratitude to the care received in the hospital. Clients who belong to the German and Ukrainian cultures may not accept their illness and may fight against the illness in them. Clients who belong to a more secular culture or are less identified by religious institutions may not accept their imminent death.

The American parents of an adopted Spanish preschooler inform the nurse that the child often stutters while speaking. Upon assessment, the nurse finds that there is no hearing impairment, brain injuries, or developmental disorders in the child. What does the nurse suspect is the cause for the stuttering?

The change in language exposure has caused stuttering. A change in language exposure may sometimes cause a child to stutter as the child takes time to master the new language. If the child is pressured to speak, there may be articulation problems. The child will be quiet and often speak less if the child is not comfortable with the new environment. If the environment is not a happy or positive one, the child will have other social and developmental issues along with speech problems.

The preschool-age client is learning sociocultural mores. What should this imply to the nurse regarding this client? The child is developing a conscience. The child is learning about gender roles. The child is developing a sense of security. The child is learning about the political process.

The child is developing a conscience. Rationale Learning the sociocultural mores of the family implies that the child is developing a conscience. This does not imply that the child is learning gender roles, developing a sense of security, or learning about the political process.

An elderly adult suffered an injury after falling down in the washroom. The primary healthcare provider performed a surgical procedure on the client and orders a blood transfusion. A family member of the client mentions that blood transfusions are not permitted in their community. What should the nurse do in order to handle the situation?

The nurse should inform the primary healthcare provider and not give blood to the client. The client or the client's family member has the right to refuse treatment and the nurse should value their beliefs and traditions. Therefore, the nurse should inform the primary healthcare provider and not perform the blood transfusion. The nurse should not wait for a court's order or explain or convince the family member to change his or her mind. The nurse should not proceed with the treatment because this may cause severe legal implications.

The nurse is overseeing a nursing student who is conducting an assessment of a client who does not speak English. No interpreter is available. Which action requires further teaching?

Using medical terminology Nurses should follow certain guidelines when interpreter is not available while assessing a client who does not understand English. Rather than using medical terminology, the nursing student should use simple, more well-known words, like "pain" instead of "discomfort." The nursing student's other actions are appropriate. Proceeding in an unhurried manner; speaking in a low, moderate voice; and pantomiming words and simple actions while verbalizing them promote effective communication.

A nurse notes that a famous client has received an incorrect dose of medication due to the malfunction of the intravenous (IV) device, but does not inform the primary healthcare provider. Instead the nurse tells a colleague that the medication could not be given due to the client's inappropriate behavior. The nurse then updates media personnel about the client's health status. What legal charges may be brought up against the nurse? Select all that apply.

slander malpractice invasion of privacy

Performance improvement focuses on analyzing and evaluating the current performance of healthcare workers in order to bring about a qualitative change. In the given situation, a new technique for performing sponge and instrument counts in the operating room is being analyzed in order to help understand the degree of qualitative change in the system. This is an example of a performance improvement. Quality improvement focuses on the continuous study and improvement of the processes of providing healthcare services to clients. Installing new call bells, conducting client teaching programs before discharge and speeding up the process of obtaining are examples of quality improvements.

true

During an assessment which client statement may indicate to the nurse that the client is experiencing spiritual distress?

"I deserve a better life than this. I don't understand why God decided to make me ill." Spiritual distress is a disturbance in a client's belief system which can cause a loss of faith and an inability to experience and integrate life's meaning and purpose. The client expressing anger at God for causing his or her illness is questioning his or her spirituality, which indicates he or she is in spiritual distress. The client searching for a divine existence, the client who expresses faith that God is with him or her, and the client who is grateful for his or her support system are showing signs of positive spiritual health.

A woman who has just delivered an infant asks to take the placenta home with her upon discharge. What is the most appropriate response by the nurse? "I'll wrap that right up for you." "I'm sorry, but you can't do that." "I'll give it to you for your husband to take home now." "I need to check the hospital protocol for our policy on that practice."

"I need to check the hospital protocol for our policy on that practice." Rationale The placenta is a part of the body and therefore contains body fluids. It must first be assessed by the healthcare provider to be sure that it is not infected and to be sure that all parts of the placenta have been accounted for. The nurse must follow hospital policy regarding the release of the placenta to the family. All necessary documentation must be signed and the policies must be followed before the release of the placenta to the family.

Which theory proposes that older adults experience a shift from a materialistic to cosmic view of the world? 1Activity theory 2Continuity theory 3Disengagement theory 4Gerotranscendence theory

-4Gerotranscendence theory Rationale: The gerotranscendence theory is a recent theory that proposes that the older adult experiences a shift in perspective with age. The person moves from a materialistic and national view of the world to a more cosmic and transcendent one. The activity theory considers the continuation of activities performed during middle age as necessary for successful aging. The continuity theory suggests that a person's personality remains stable and behavior becomes more predictable as people age. The disengagement theory states that aging individuals withdraw from customary roles and engage in more introspective, self-focused activities.

A client who only speaks Spanish is being cared for at a hospital in which nursing personnel only speak English. What communication technique would be appropriate for the nurse to use when discussing healthcare decisions with the client? Correct1 Contact an interpreter provided by the hospital. 2 Contact the client's family member to translate for the client. 3 Communicate with the client using Spanish phrases the nurse learned in a college course. 4 Communicate with the client with the use of a hospital-approved Spanish dictionary.

1

A client with the diagnosis of borderline personality disorder is manipulative and uses this behavior to get cigarettes from other clients. One day the client begins to badger another client. What should the nurse say while removing the client from the area? Correct1 "You must leave people alone; this behavior is unacceptable." 2 "There will be consequences if you do not stop annoying people." 3 "Tell me how you feel when you are exerting control over people." 4 "I'm surprised that you're still bothering people; you seemed to have improved lately."

1

A depressed client tells a nurse, "I want to die." What is the most therapeutic response by the nurse? Correct1 "You would rather not live." 2 "You're not alone in feeling this way." 3 "When was the last time you felt this way?" 4 "Do you believe that there's life after death?"

1

A mother brings her 9-month-old infant to the clinic. The nurse is familiar with the mother's culture and knows that belly binding to prevent extrusion of the umbilicus is a common practice. The nurse accepts the mother's cultural beliefs but is concerned for the infant's safety. What variation of belly binding does the nurse discourage? Correct1 Coin in the umbilicus 2 Tight diaper over the umbilicus 3 Binder that encircles the umbilicus 4 Adhesive tape across the umbilicus

1

A nonviolent client on the psychiatric unit suddenly refuses to take the prescribed antipsychotic medication. What should the nurse do? Correct1 Honor the client's decision and document the behavior and all interventions. 2 Use an authoritarian approach to induce the client to take the prescribed medication. 3 Call the primary healthcare provider and request that the client be discharged against medical advice. 4 Start proceedings to have the client declared incompetent and seek a court order permitting medication.

1

A nurse is accompanying a client with a diagnosis of anxiety disorder who is pacing the halls and crying. When the client's pacing and crying worsen, the nurse suddenly feels uncomfortable and experiences a strong desire to leave. What is the most likely reason for what the nurse is experiencing? Correct1 An empathic communication of anxiety 2 A fear of the client becoming assaultive 3 A desire to go off duty after a busy workday 4 An inability to tolerate any more bizarre behavior

1

A nurse is caring for a client who had a nephrectomy because of cancer of the kidney. Which factor will influence the client's ability to deep breathe and cough postoperatively? Correct1 Location of the surgical incision 2 Increased anxiety about the prognosis 3 Inflammatory process associated with surgery 4 Pulmonary congestion from preoperative medications

1

A nurse is working in a daycare center with clients who have cognitive impairments. What does the nurse expect of a client in the middle stages of dementia? Correct1 Ability to recall events from the past 2 Ability to cope effectively with anxiety 3 Ability to follow a simple schedule without help 4 Memory of what was eaten on the previous day

1

A nurse is working with a child who was physically abused by a parent. What is the most important goal for this family? Correct1 The child will live in a safe environment. 2 The parents will use verbal discipline effectively. 3 The family will feel comfortable in its relationship with the counselor. 4 The parents will gain an understanding of their abusive behavior patterns.

1

A nurse understands that value clarification is a technique useful in therapeutic communication because initially it helps clients do what? Correct1 Become aware of their personal values 2 Gain information related to their needs 3 Make correct decisions related to their health 4 Alter their value systems to make them more socially acceptable

1

A psychiatric nurse is hired to work in the psychiatric emergency department of a large teaching hospital. While reviewing the manuals, the nurse reads, "People with mental health emergencies shall be triaged within 5 minutes of entering the emergency department." What does the nurse consider this statement to represent? Correct1 Hospital policy 2 Standard of care 3 Hospital procedure 4 Mental Health Bill of Rights

1

A resident in a nursing home recently immigrated to the United States (Canada) from Italy. How does the nurse plan to provide emotional support? Correct1 By offering choices consistent with the client's heritage 2 By assisting the client in adjusting to American culture 3 By ensuring that the client understands American beliefs 4 By correcting the client's misconceptions about appropriate health practices

1

After a child's visit to a healthcare provider, the parent tells the nurse, "I'm so upset! The doctor prescribed an antidepressant!" What is the best response by the nurse? Correct1 "Tell me more about what's bothering you." 2 "Weren't you told why your child needs an antidepressant?" 3 "You need to speak with the healthcare provider about your concerns." 4 "Are you sure it's an antidepressant and not a drug for attention deficit disorder?"

1

The nurse is caring for a client 4 days after the client was admitted to the hospital with burns on the trunk and arms. The nurse collaborates with the dietician to develop a dietary plan for the following day. Which plan will the nurse follow? Correct1 High caloric intake, liberal potassium intake, and 3 g protein/kg/day 2 High caloric intake, restricted potassium intake, and 1 g protein/kg/day Incorrect3 Moderate caloric intake, liberal potassium intake, and 3 g protein/kg/day 4 Moderate caloric intake, restricted potassium intake, and 1 g protein/kg/day

1

The nurse is caring for a client with a diagnosis of diabetic ketoacidosis. Which arterial blood gas results are associated with this diagnosis? Correct1 pH: 7.28; PCO2: 28; HCO3: 18 2 pH: 7.30; PCO2: 54; HCO3: 28 3 pH: 7.50; PCO2: 49; HCO3: 32 4 pH: 7.52; PCO2: 26; HCO3: 20

1

The nurse is teaching a client newly diagnosed with diabetes about the importance of glucose monitoring. Which blood glucose levels should the nurse identify as hypoglycemia? Correct1 68 mg/dL (3.8 mmol/L) 2 78 mg/dL (4.3 mmol/L) 3 88 mg/dL (4.9 mmol/L) 4 98 mg/dL (5.4 mmol/L)

1

What does a nurse recall that language development in the autistic child resembles? Correct1 Echolalia 2 Stuttering 3 Scanning speech 4 Pressured speech

1

What is the color of a client's wound caused by skin tears? Correct1 Red 2 Gray 3 Black 4 Yellow

1

What is the most appropriate long-term goal for a client experiencing dysfunctional grieving after the death of a spouse? Correct1 Resuming previously enjoyed activities 2 Eating at least two meals a day with another person 3 Decreasing negativistic thinking about other people 4 Relocating to a state in which other family members reside

1

When leading the first session of a newly formed group of clients in a mental health clinic, the nurse notes that group members frequently assume self-serving roles. What does the nurse understand about this? Correct1 Early group development involves these behaviors. 2 Some group members will need to be placed in another group. Incorrect3 Certain group members may be emerging to control attention seekers. 4 The group is attempting to reconcile conflicting viewpoints among its members.

1

When the nurse is managing the care of an acutely depressed client, which intervention demonstrates that the nurse recognizes the client's fundamental mental health need? Correct1 Role modeling a hopeful attitude regarding life and the future 2 Sharing that life has presented depressing situations for all of us at times 3 Devoting time with the client and trying to focus on happy, positive memories Incorrect4 Identifying the client's personal weaknesses and designing interventions to strengthen them

1

Which behavior is seen in children at the undifferentiated stage of spiritual development, as propounded by Fowler? Correct1 Children have no concept of right or wrong to guide their behaviors. 2 Children imitate the religious behaviors without comprehending any meaning. 3 Children reason and question some of the established parental religious standards. 4 Children have a reverence for religious matters and are able to articulate their faith.

1

Which type of laser is used in the treatment of vascular and other pigmented lesions? Correct1 Argon 2 Gold vapors Incorrect3 Neodymium 4 Carbon dioxide

1

A nurse notices that a client is in spiritual distress. Which nursing action establishes the nurse as a caregiver? 1The nurse provides therapeutic treatment to the client. 2The nurse teaches the client about signs of spiritual distress. 3The nurse communicates the wishes of the client to family members. 4The nurse collaborates with the agency chaplain to pursue the best treatment plan.

1The nurse provides therapeutic treatment to the client.

A 24-year-old woman states that she no longer enjoys any of the activities that she once found fun and pleasurable, such as socializing, sports, and hobbies. What term should the nurse use to describe this condition? 1 Anergia Correct2 Anhedonia 3 Grandiosity 4 Learned helplessness

2

A 54-year-old has demonstrated increasing forgetfulness, irritability, and antisocial behavior. After the person is found disoriented and semi-naked while walking down a street, the diagnosis of dementia of the Alzheimer type is made. The client expresses fear and anxiety upon admission to a long-term care facility. What is the best nursing intervention in light of the client's diagnosis? 1 Exploring the reasons for the concerns Correct2 Reassuring the client with the frequent presence of staff members 3 Providing the client with a written schedule of planned interactions 4 Explaining to the client why the admission to the facility is necessary

2

A 68-year-old client who has metastatic carcinoma is told by the practitioner that death will occur within a month or two. Later the nurse enters the client's room and finds the client crying. Before responding, which factor should the nurse consider? 1 Crying relieves depression and helps the client face reality. Correct2 Crying releases tension and frees psychic energy for coping. 3 Nurses should not interfere with a client's behavior and defenses. 4 Accepting a client's tears maintains and strengthens the nurse-client bond.

2

A Hispanic nurse is celebrating a quinceañera for her 15-year-old daughter. What is the appropriate action of the culturally competent nurse manager? 1 Make a joke about the cultural ritual. Correct2 Allow the nurse to take time off for this celebration. 3 Ask the nurse not to put so much value in cultural rituals. 4 Suggest that the nurse not invite other staff members to the occasion.

2

A client with a generalized anxiety disorder is hospitalized. The nurse determines that what environment would be conducive to reducing emotional stress and providing psychologic safety for this client? 1 One in which needs are met Correct2 One in which realistic limits and controls are set 3 One in which the client's requests are met promptly 4 One in which the client's environment is kept neat and orderly

2

A multigravida of Asian descent weighs 104 lb (47.2 kg), having gained 14 pounds (6.4 kg) during the pregnancy. On her second postpartum day, the client's temperature is 99.2° F (37.3° C). She has had poor dietary intake since admission. What should the nurse do? 1 Ask the nursing supervisor to discuss this with the healthcare provider. Correct2 Encourage the family to bring in special foods preferred in their culture. 3 Order a high-protein milkshake as a between-meal snack to stimulate her appetite. 4 Explain to the family that the dietitian plans nutritious meals that the client should eat.

2

A nurse identifies premature ventricular complexes (PVCs) on a client's cardiac monitor. What does the nurse conclude that these complexes are a sign of? 1 Atrial fibrillation Correct2 Cardiac irritability 3 Impending heart block Incorrect4 Ventricular tachycardia

2

A nurse in the mental health clinic concludes that a client is using confabulation when the client does what? 1 The flow of thoughts is interrupted. Correct2 Imagination is used to fill in memory gaps. 3 Speech flits from one topic to another with no apparent meaning. 4 Connections between statements are so loose that only the speaker understands them.

2

A nurse is assessing a client who has come to a walk-in mental health clinic. Which statement supports the assessment that the client is experiencing a crisis? 1 "I have these feelings of uneasiness. They come and go." Correct2 "Nothing I try works. Everything just keeps getting worse." 3 "Things have been building up slowly. I don't know what's causing it." 4 "I feel tense and irritable. When I concentrate on my work, I feel better."

2

After reviewing the client's laboratory reports, the physician concludes that the client has primary hypofunction of the adrenal gland. Which clinical manifestation is likely to be observed in that client? 1 Edema at extremities Correct2 Uneven patches of pigment loss 3 Reddish-purple stretch marks on the abdomen 4 "Buffalo hump" between shoulders on the back

2

An Asian client arrives at the mental health clinic with symptoms of anxiety and panic. While speaking with the client, the nurse notes that the client makes very little eye contact. What does this assessment data suggest? 1 Shyness Correct2 Cultural variation 3 Symptom of depression 4 Shame regarding treatment

2

An older adult, accompanied by family members, is admitted to a long-term care facility with symptoms of dementia. What initial statement by the nurse during the admission procedure would be most helpful to this client? 1 "You're a little disoriented now, but don't worry. You'll be all right in a few days." Correct2 "Don't be afraid. I'm your nurse, and everyone here in the hospital is here to help you." 3 "I'm the nurse on duty today. You're in the hospital. Your family can stay with you for a while." Incorrect4 "Let me introduce you to the staff here first. In a little while I'll get you acquainted with our unit routine."

2

An older resident in a nursing home who has a diagnosis of dementia hoards leftover food from the meal tray and other seemingly valueless articles and stuffs them into pockets "so the others won't steal them." What should the nurse plan to do? 1 Remove the resident's unsafe and soiled articles during the night. Correct2 Give the resident a small bag in which to place selected personal articles and food. 3 Explain to the resident why the nursing home's policy for cleanliness and safety must be followed. 4 Explain to the resident that the staff is required to keep harmful objects out of reach in the resident's closet.

2

Obesity in children is an ever-worsening problem. What concept should a nurse consider when caring for school-aged children who are obese? 1 Enjoyment of specific foods is inherited. Correct2 There are familial influences on childhood eating habits. 3 Childhood obesity is usually not a predictor of adult obesity. 4 Children with obese parents are destined to become obese themselves.

2

The nurse is caring for a client with vascular dementia. What does the nurse identify as the cause of this problem? 1 A long history of inadequate nutrition Correct2 Disruptions in cerebral blood flow, resulting in thrombi or emboli 3 A delayed response to severe emotional trauma in early adulthood 4 Anatomical changes in the brain that produce acute, transient symptoms

2

The nurse should understand the effects of internal and external variables to plan and deliver individualized care. Which variable is an internal variable? 1 Family practices Correct2 Emotional factors 3 Cultural background 4 Socioeconomic factors

2

The nurse teaches a client methods of coping with anger. The nurse concludes that the client has learned the most effective method when the client states that the client will do what when angry? 1 Go for a long jog. Correct2 Talk about the anger. 3 Go to the basement to scream. 4 Concentrate on what caused the anger.

2

The primary healthcare provider prescribed a diagnostic study with contrast medium for an older adult who has an endocrine disorder. Which assessment result should the nurse check before the study? 1 Urinary pH Correct2 Serum creatinine 3 Urinary creatinine Incorrect4 Creatinine clearance

2

The serum potassium level of a client who has diabetic ketoacidosis is 5.4 mEq/L (5.4 mmol/L). What would the nurse expect to see on the ECG tracing monitor? 1 Abnormal P waves and depressed T waves Correct2 Peaked T waves and widened QRS complexes 3 Abnormal Q waves and prolonged ST segments 4 Peaked P waves and an increased number of T waves

2

A client who has a phobia about dogs is about to begin systematic desensitization. The client asks what the treatment will involve. What is the best response by the nurse? 1 "You'll be exposed to dogs until you no longer feel anxious." 2 "Rewards will be given when you don't become anxious around dogs." Correct3 "Your contact with dogs will be increased, and we'll teach you relaxation techniques." 4 "We'll be engaging in detailed discussions to help you identify what caused your phobia."

3

A client with a history of chronic alcoholism is admitted to the mental health unit. What does the nurse identify as the cause of a client's use of confabulation? 1 Ideas of grandeur 2 Need to get attention Correct3 Marked loss of memory 4 Difficulty accepting the truth

3

A mother brings her 7-year-old son into an outpatient clinic for a follow-up appointment. The mother appears angry and agitated with the boy. Looking at the boy's medical chart, the nurse notes that the boy has a diagnosis of encopresis. What is the primary symptom of encopresis? 1 Practicing self-mutilation 2 Practicing self-induced vomiting Correct3 Passing feces either voluntarily or involuntarily into inappropriate places 4 Passing urine either voluntarily or involuntarily into inappropriate places

3

A nurse has just finished reviewing how anesthesia will be used during a vaginal birth for a client with class I heart disease. What type of anesthesia does the client discuss that indicates to the nurse that the teaching was effective? 1 Spinal 2 General Correct3 Epidural 4 Perinea

3

A nurse hired to work in a metropolitan hospital provides services for a culturally diverse population. One of the nurses on the unit says it is the nurses' responsibility to discourage "these people" from bringing all that "home medicine stuff" to their family members. Which response by the recently hired nurse is most appropriate? 1 "Hospital policies should put a stop to this." 2 "Everyone should conform to the prevailing culture." Correct3 "Nontraditional approaches to health care can be beneficial." 4 "You are right because they may have a negative impact on people's health."

3

A nurse is assessing a client who has come to a walk-in mental health clinic. Which statement supports the assessment that the client is experiencing a crisis? 1 "I have these feelings of uneasiness. They come and go." Correct2 "Nothing I try works. Everything just keeps getting worse." 3 "Things have been building up slowly. I don't know what's causing it." 4 "I feel tense and irritable. When I concentrate on my work, I feel better."

3

A registered nurse is teaching a nursing student about the importance of values in nursing practice. Which information provided by the registered nurse is appropriate? Select all that apply. 1 "People may consider strong values as opinions." 2 "Evaluate a client's values and beliefs in terms of your own values." 3 "Values vary among clients and develop and change over time." 4 "The values that an individual holds reflect cultural and social influences." 5 "To discuss differences in opinions and values, the nurse should be clear about his or her own values."

3 "Values vary among clients and develop and change over time." 4 "The values that an individual holds reflect cultural and social influences." 5 "To discuss differences in opinions and values, the nurse should be clear about his or her own values."

As the nurse is discussing psychiatric care with an older adult client, the client says, "When I was growing up I was taught to accept my lot in life and not complain. I'm proud of the fact that despite my issues I can still function independently. I don't want to be just put away." The nurse understands that the factors that influence the client's mental health are examples of what? 1Setting of care 2Anxiety disorder 3Attitudes and beliefs 4Cultural and ethnic disparities

3 Attitudes and beliefs

A nurse is caring for a client who believes that surgery contaminates the soul unless proper cleansing rituals are performed. Which nursing activity adheres to the Quality and Safety Education for Nurses (QSEN) competency patient-centered care? 1 Evaluating the effects of complying with a client's requests on his or her psychological health 2 Improving therapeutic communication skills in order to minimize the risk of emotional trauma 3 Coordinating with appropriate spiritual practitioners to help perform the cleansing ritual before the client's surgery 4 Conducting extensive research about the ritualistic practices of the client's community to formulate a care plan

3 Coordinating with appropriate spiritual practitioners to help perform the cleansing ritual before the client's surgery

A nurse is helping a client who observes the traditional Jewish dietary laws to prepare a dietary menu. What considerations should the nurse make? 1Eating beef and veal is prohibited. 2Consumption of fish with scales is forbidden. 3Meat and milk at the same meal are forbidden. 4Consuming alcohol, coffee, and tea are prohibited.

3 Meat and milk at the same meal are forbidden.

A pregnant client whose first child has Down syndrome is about to undergo amniocentesis. The client tells the nurse that she does not know what she will do if this fetus has the same diagnosis. The client asks the nurse, "Do you think abortion is the same as killing?" How should the nurse respond? 1"Some people think that that's what an abortion is." 2"No, I don't think so, but it's your decision to make." 3"I really can't answer that question. Are you ambivalent about abortion?" 4"I don't want to answer that question at this time. How do you feel about it?"

3"I really can't answer that question. Are you ambivalent about abortion?"

A nurse from the pediatric clinic who is strongly opposed to any chemical or mechanical method of birth control is asked to work in the family planning clinic. What is the most professional response that this nurse could give to the requesting supervisor? 1"I will go, but it is against my beliefs and values." 2"I won't do it, because I do not believe in birth control at all." 3"I would prefer another assignment that is not contrary to my beliefs." 4"I will have to stress that the rhythm method is the method of choice."

3"I would prefer another assignment that is not contrary to my beliefs."

A nurse in a hospice program cares for clients and family members who are coping with imminent loss. What is the most important factor in predicting a person's potential reaction to grief? 1 Family interactions 2 Social support system 3 Emotional relationships Correct4 Earlier experiences with grief

3,4

Which drugs may cause an increase in the serum clozapine level? Select all that apply. 1 Rifampin 2 Phenytoin Correct3 Ketoconazole Correct4 Erythromycin 5 Bromocriptine

3,4

Which hormones does the nurse state are released by the hypothalamus? Select all that apply. 1 Follicle-stimulating hormone (FSH) 2 Thyroid-stimulating hormone (TSH) Correct3 Melanocyte-inhibiting hormone (MIH) Correct4 Corticotropin-releasing hormone (CRH) Correct5 Growth hormone-releasing hormone (GHRH)

3,4,5

A client had a colostomy surgery and is learning how to care for the skin around the stoma. Which information should the nurse include in the teaching plan for this client? Incorrect1 "Cut an opening about ⅓ inch (0.85 cm) larger than the stomal pattern." 2 "Avoid the use of soap and other irritating agents." 3 "Eat yogurt and drink buttermilk and parsley." Correct4 "Empty the pouch before it is one-third full."

4

A nurse is with the parents of a 3-year-old child who has just died. What is the most therapeutic question for the nurse to ask the parents? 1 "Do you feel ready to consent to an autopsy?" 2 "Have you made a decision about organ donation?" 3 "Would you like to talk about how you'll tell your other children?" Correct4 "Can I be of any help with traditional practices that are important to you?"

4

An autistic toddler is sitting in a corner, rocking and spinning a top. How can the nurse be most therapeutic when approaching this toddler? 1 Holding the toddler to provide a sense of security 2 Stroking the toddler's arm gently to gain the child's attention 3 Waiting for the toddler to make the initial contact before moving close Correct4 Sitting with the toddler while watching the spinning top to provide a nonintrusive presence

4

An older adult is admitted for evaluation of anemia and unsteady gait. While obtaining a health history, the nurse notes that the client seems to make up stories to fill in for memory lapses. How should the nurse document what the client is doing? 1 Lying 2 Denying 3 Fantasizing Correct4 Confabulating

4

Before effectively responding to a sexually abused victim on the phone, it is essential that the nurse in the rape crisis center do what? 1 Get the client's full name and address. 2 Call for assistance from the psychiatrist. 3 Know some myths and facts about sexual assault. Correct4 Be aware of any personal bias about sexual assault.

4

Imipramine, 75 mg three times per day, is prescribed for a client. What nursing action is appropriate when this medication is being administered? 1 Telling the client that barbiturates and steroids will not be prescribed 2 Warning the client not to eat cheese, fermented products, and chicken liver 3 Monitoring the client for increased tolerance and reporting when the dosage is no longer effective Correct4 Having the client checked for increased intraocular pressure and teaching about symptoms of glaucoma

4

What should a nurse include in the initial plan of care for a client with the long-standing obsessive-compulsive behavior of handwashing? 1 Determining the purpose of the ritualistic behavior Incorrect2 Limiting the time allowed for the ritualistic behavior 3 Suggesting a symptom-substitution technique to refocus the ritualistic behavior Correct4 Developing a routine schedule of activities to reduce the need for the ritualistic behavior

4

Which drug can be prescribed to a 30-week-pregnant woman to promote maturity of the lungs in the fetus? 1 Methylergonovine 2 Clomiphene 3 Dinoprostone Correct4 Betamethasone

4

Which skin color in a client indicates an increased urochrome level? 1 Red 2 Blue 3 Reddish blue Correct4 Yellow-orange

4

Which technology would the nurse use to reduce chronic ulcers by removing fluids from the wound? 1 Electrical stimulation 2 Topical growth factors 3 Hyperbaric oxygen therapy Correct4 Negative pressure wound therapy

4

Which type of allergic skin condition in a client is associated with immunological irregularity, asthma, and allergic rhinitis? 1 Urticaria 2 Psoriasis 3 Acne vulgaris Correct4 Atopic dermatitis

4

The nurse is assessing a Latino-Caribbean client who was brought to the hospital by family members. The family reports the client started crying, shouting, trembling, had uncontrolled jerking of the extremities, and then fell into a trance-like state. What condition does the nurse suspect? Bulimia nervosa Anorexia nervosa Shenjing shuairuo Ataque de nervios

Ataque de nervios Rationale Ataque de nervios is a Latino-Caribbean culture-bound syndrome that usually happens in response to specific stressors. This culture-bound syndrome is characterized by crying, uncontrollable spasms, trembling, shouting, dissociation, and trance-like states. Bulimia nervosa and anorexia nervosa are culture-bound syndromes in the form of eating disorders, but they are not characterized by crying, spasms, and shouting. Shenjing shuairuo is not associated with the Latino-Caribbean culture; instead, it is associated with Chinese culture.

How can the lines of communication be improved in a healthcare organization during the process of delegation? By considering all aspects of client care By selecting experienced nursing assistants as delegatees By appreciating and valuing each other's cultural perspectives By selecting a delegatee having similar strengths as that of the delegator

By appreciating and valuing each other's cultural perspectives Rationale The lines of communication in a healthcare organization can be improved by appreciating and valuing each other's cultural perspectives, which balances strengths between the delegator and delegatee and improves client care outcomes. Considering all aspects of client care ensures that all of the client care needs are addressed. Selecting experienced nursing assistants as delegatees increases the chances of the delegatee to adapt to changing situations. Selecting a delegatee having similar strengths as that of the delegator may decrease the lines of communication because the delegatee might do the task of the delegator.

A resident in a nursing home recently immigrated to the United States (Canada) from Italy. How does the nurse plan to provide emotional support? By offering choices consistent with the client's heritage By assisting the client in adjusting to American culture By ensuring that the client understands American beliefs By correcting the client's misconceptions about appropriate health practices

By offering choices consistent with the client's heritage Rationale Adherence to a plan of care is enhanced by the nurse's providing choices consistent with the client's cultural beliefs and practices. The nurse's cultural or personal beliefs and biases should not influence or interfere with the implementation of appropriate care. Helping the client adjust to American culture is not the priority at this time; care should be adapted to the client's needs and culture. The person's cultural practices should not be addressed unless they are detrimental to the person's health.

A hospice nurse is caring for a dying client and the client's family members during the developing awareness stage of grief. What is the most important thing about the family that the nurse should assess before providing care? Cohesiveness Educational level Cultural background Socioeconomic status

Cultural background Rationale During the developing awareness stage of grief the degree of anguish experienced or expressed is influenced by the cultural background of the individual and family. Although cohesiveness does enter into the grief process, it is not as important in the developing awareness stage as cultural background is. Educational level has no relationship to the grieving process. Socioeconomic status is not a defining factor in how a family will respond to the loss of a loved one.

A pregnant immigrant notices cultural differences in the way that pregnant women are cared for where she now lives. Which component of cultural competence is being demonstrated when the nurse motivates the immigrant to accept these differences? Cultural desire Cultural awareness Cultural knowledge Cultural encounters

Cultural desire Rationale The nurse is using cultural desire as a part of cultural competence. This component is related to motivation and commitment towards the care of an individual. Through this, an immigrant may become open to cultural differences and accept them. Cultural awareness is an in-depth self-examination of backgrounds and recognizing biases and prejudices. Cultural knowledge is a comparative study about the beliefs and care practices of other cultures. Cultural encounter is about transcultural interactions for effective communication and development.

The nurse leader states, "The people in rural America dress and act differently from those in urban centers." What concept describes this statement? Acculturation Ethnocentrism Cultural imposition Cultural marginality

Cultural marginality Rationale Cultural marginality is defined as situations and feelings of passive betweenness when people exist between two different cultures. refers to adapting to a particular culture. It is a process by which a person becomes a competent participant in the dominant culture. Ethnocentrism refers to the belief that one's own ways are the best, most superior, or preferred ways to act, believe, or behave. Cultural imposition is defined as the tendency of an individual or group to impose their values, beliefs, and practices on another culture for varied reasons.

An Asian client arrives at the mental health clinic with symptoms of anxiety and panic. While speaking with the client, the nurse notes that the client makes very little eye contact. What does this assessment data suggest? Shyness Cultural variation Symptom of depression Shame regarding treatment

Cultural variation Rationale As a show of respect, people in Asian cultures tend to make little eye contact, particularly with people perceived as authority figures. A lack of eye contact may connote shyness in some clients, but further assessment is needed. A lack of eye contact may suggest a depressed mood; however, there is no indication of depression in this client. A lack of eye contact may indicate shame or low self-esteem in the American culture; however, it is important not to make this same interpretation of behavior for someone from another culture.

A 5-year-old child who is newly arrived from Latin America attends a nursery school where everyone speaks English. The child's mother tells the nurse that her child is no longer outgoing and has become very passive in the classroom. What is the probable reason for the child's behavior? Culture shock Social immaturity Experience of discrimination Lack of interest in school activities

Culture shock Rationale The child learned to think and solve problems in a different culture and language and may feel helpless in the new classroom. There are no data to indicate that social immaturity, discrimination, or lack of interest is the precipitating factor for the child's behavior.

Why is it important for a nurse in the prenatal clinic to provide nutritional counseling to all newly pregnant women? Most weight gain is caused by fluid retention. Different cultural groups favor different essential nutrients. Dietary allowances should not increase throughout pregnancy. Pregnant women must adhere to a specific pregnancy dietary regimen.

Different cultural groups favor different essential nutrients. Rationale The nurse should become informed regarding the cultural eating patterns of clients so that foods containing the essential nutrients that are part of these dietary patterns may be included in the diet. Fluid retention is only one component of weight gain; growth of the fetus, placenta, breasts, and uterus also contributes to weight gain. The need for calories and nutrients increases during pregnancy. Pregnancy diets are not specific; they are composed of the essential nutrients.

A multigravida of Asian descent weighs 104 lb (47.2 kg), having gained 14 pounds (6.4 kg) during the pregnancy. On her second postpartum day, the client is withdrawn and eating very little from the meals provided. Which intervention is most important for the nurse to implement? Report these findings to the healthcare provider. Encourage the family to bring in special foods preferred in their culture. Order a high-protein milkshake to supplement between meals. Call the dietitian to work with client to plan high calorie meals for the client to eat.

Encourage the family to bring in special foods preferred in their culture. Rationale In family-centered childbearing, care should be adapted to the client's cultural needs and preferences whenever possible. Discussing the problem with the healthcare provider is the nurse's responsibility but will not address the client's preferences. Ordering a high-protein milkshake as a between-meal snack may offer the client an option but is unlikely to meet the cultural preferences. Having the dietitian assist with planning meals does not address the underlying problem.

What does the professional nurse consider to be the center of decision-making when providing client care? Ethics of care Nursing skills Analytical skills Research based practice

Ethics of care Rationale A professional nurse always follows the ethics of care and considers caring to be the center of decision-making. The nurse must know what behavior is ethically appropriate while caring for a client. A nurse's effectiveness in performing tasks is important to client care; however, client satisfaction comes from the effective dimension of care. Because ethics of care are unique to each client, the nurse should not base decision-making only on analytical skills. The nurse should not provide client care based only on intellectual principles or research knowledge. Caring is the most important factor because it considers client preferences and values.

The nurse has just arrived in the unit for her shift at the healthcare facility. There are two new clients admitted to the unit. What should the nurse do first to collect the first set of information about the clients assigned to his or her care?

Participate in the bedside rounds The nurse should participate in bedside rounds with the healthcare team from the previous shift. The nurse who is completing care for one shift prepares the change-of-shift report to communicate client details to the nurse in the next shift. These bedside rounds provide patient-centered care as the nurse shares information about the client's condition, status of problems, and treatment plan for the next shift. The nurse can meet the client's family after obtaining firsthand information from the nurse completing the shift. The nurses review the client's medical reports and discuss treatment plans for the next shift after completing the bedside rounds. The nurse may meet the client during bedside rounds or after obtaining the handover report.

What does a public health nurse expect to encounter when working with families raised in a culture of poverty?

Powerlessness relative to changing their situation Powerlessness is a characteristic feeling among people in the culture of poverty, which tends to erode their hope for change. People in the culture of poverty usually require immediate gratification because they do not have enough faith in the future to delay gratification. Pessimism, not optimism, about changing a lifestyle is more common in these families. There is not sufficient evidence to indicate that poor people feel shame for their situation or that they are inadequate parents.

While supervising a smallpox vaccination program, a nurse manager observes a nurse cleansing the arm of a client with an alcohol swab before giving the vaccination. What should the nurse manager's first reaction be?

Stop the nurse from giving the injection *Alcohol deactivates the smallpox vaccine*. Cleansing of the arm should not be done before the immunization is given unless the arm is dirty; if dirty, only water should be used to cleanse the site. Observation is insufficient; the nurse manager must intervene to ensure that the vaccine is given using the correct technique. Povidone-iodine will deactivate the smallpox vaccine. The site should be dry before administering the vaccine.

While talking with a 60-year-old client, the nurse learns that the client emigrated 15 years ago from China and likes to live independently away from the client's grown children. The client eats only Chinese foods at home. What should the nurse infer from these findings?

The client has undergone biculturalism. When different cultures encounter other cultures, they tend to adapt the cultural beliefs and psychology of each other's culture. The people of Chinese culture feel that their children are responsible for caring for them in old age. Therefore when Chinese people adapt some characteristics of American culture but prefer to eat only Chinese foods, it indicates that the client has undergone biculturalism. Assimilation is a cultural process by which a person can give up their original identity and develop a new cultural identity. In the process of acculturation, both the host culture and the original culture are changed as a result of reciprocal influences. In the process of enculturation, the person learns norms, values, and behaviors of their own culture, similar to socialization.

A nurse notices that a client is in spiritual distress. Which nursing action establishes the nurse as a caregiver?

The nurse provides therapeutic treatment to the client A nurse serves as a caregiver by meeting all the health care requirements of the client by providing measures that restore a client's emotional, spiritual, and social well-being. In the given scenario, the nurse provides therapeutic treatment to the client as a caregiver. As an educator, the nurse teaches the client about the signs of spiritual distress. As a client's advocate, the nurse communicates the wishes of the client to family members. The nurse follows the principle of accountability by collaborating with the agency chaplain to pursue the best treatment plan.

The Surgeon General used the data from the 2000 census classification system to identify disparities in mental health care along racial-ethnic lines. What is the secondary use of this data?

To determine when and how the health care needs of the ethnic populations are being met The census classification system categorized individuals according to racial and ethnic descriptions. In addition to identifying health disparities, recording these classifications helps to determine when and how the health care needs of ethnic populations are being met. Nurses should practice culturally relevant nursing in order to meet the needs of culturally diverse clients of a specific ethnic group; the census has nothing to do with this. Because each racial group contains multiple ethnic cultures, the census does not succeed in identifying all of them in the United States, and it doesn't include them all as options. Although the census helps identify health disparities, it does not attempt to examine and determine why they exist.


Related study sets

Med surg success integumentary disorders comprehensive exam

View Set

A&P 231 ccc Classification and structure of bones and cartilage

View Set

Real Estate Final Exam Attempt Rd 2

View Set

Chapter 17, 18, 19: Preoperative, Intraoperative, Postoperative

View Set

William Wordsworth: ''I Wandered Lonely As A Cloud''

View Set